61
8/20/2019 Budget Methodologies http://slidepdf.com/reader/full/budget-methodologies 1/61 Question 1 - CMA 691 3-11 - Budget Methodologies When budgeting, the items to be considered by a manufacturing firm in going from a sales quantity budget to a production budget would be the  A. Expected change in the quantity of work-in-process inventories. B. Expected change in the quantity of finished goods and raw material inventories. C. Expected change in the availability of raw material without regard to inventory levels. D. Expected change in the quantity of finished goods and work-in-process inventories.  A. The expected change in the level of finished goods inventories should be considered as well in the development of the production budget. B. The expected change in the work-in process inventory level, not the raw materials inventories levels should be considered in determining the production budget. C. The existing levels of inventories of work-in-process and finished goods are both considered in the determination of production levels. D. To decide what quantity should be manufactured during a period given the amount of sales for the period, the levels of finished goods inventory and work-in-process inventories should also be considered. Question 2 - CMA 692 3-27 - Budget Methodologies Esplanade Company, which has the following historical pattern for its credit sales: 70% collected in month of sale 15% collected in the first month after sale 10% collected in the second month after sale 4% collected in the third month after sale 1% uncollectible The sales on open account have been budgeted for the last 6 months of the year as shown below. July $60,000  August 70,000 September 80,000 October 90,000 November 100,000 December 85,000 The estimated total cash collections during October from accounts receivable would be  A. $21,400. B. $84,400. C. $86,700. D. $63,000.  A. This amount does not include cash collected in October from October's sales but only the collections from July,  August and September sales collected in October. B. Cash collections in October represent cash collections from sales made in October as well as collections from sales made in previous months according to the cash collection schedule, as follows: October sales × .70 = $90,000 × .70 = $63,000 Part 1 : 07/28/10 08:28:23 (c) HOCK international, page 1

Budget Methodologies

  • Upload
    arief

  • View
    216

  • Download
    0

Embed Size (px)

Citation preview

Page 1: Budget Methodologies

8/20/2019 Budget Methodologies

http://slidepdf.com/reader/full/budget-methodologies 1/61

Question 1 - CMA 691 3-11 - Budget Methodologies

When budgeting, the items to be considered by a manufacturing firm in going from a sales quantity budget to aproduction budget would be the

 A. Expected change in the quantity of work-in-process inventories.B. Expected change in the quantity of finished goods and raw material inventories.C. Expected change in the availability of raw material without regard to inventory levels.D. Expected change in the quantity of finished goods and work-in-process inventories.

 A. The expected change in the level of finished goods inventories should be considered as well in the developmentof the production budget.

B. The expected change in the work-in process inventory level, not the raw materials inventories levels should beconsidered in determining the production budget.

C. The existing levels of inventories of work-in-process and finished goods are both considered in the determinationof production levels.

D. To decide what quantity should be manufactured dur ing a period given the amount of sales for the

period, the levels of finished goods inventory and work-in-process inventories should also be considered.

Question 2 - CMA 692 3-27 - Budget Methodologies

Esplanade Company, which has the following historical pattern for its credit sales:

70% collected in month of sale

15% collected in the first month after sale

10% collected in the second month after sale

4% collected in the third month after sale

1% uncollectible

The sales on open account have been budgeted for the last 6 months of the year as shown below.

July $60,000

 August 70,000

September 80,000

October 90,000

November 100,000

December 85,000

The estimated total cash collections during October from accounts receivable would be

 A. $21,400.B. $84,400.C. $86,700.D. $63,000.

 A. This amount does not include cash collected in October from October's sales but only the collections from July, August and September sales collected in October.

B.

Cash collections in October represent cash collections from sales made in October as well as collections

from sales made in previous months according to the cash collection schedule, as follows:

October sales × .70 = $90,000 × .70 = $63,000

Part 1 : 07/28/10 08:28:23

(c) HOCK international, page 1

Page 2: Budget Methodologies

8/20/2019 Budget Methodologies

http://slidepdf.com/reader/full/budget-methodologies 2/61

September sales × .15 = $80,000 × .15 = $12,000

 August sales × .10 = $70,000 × .10 = $7,000

Ju ly sales × .04 = $60,000 × .04 = $2,400

Total collections = $63,000 + $12,000 + $7,000 + $2,400 = $84,400

C. This is amount of total cash collected in December. See the correct answer for a complete explanation.

D. This is amount of cash collected in October from October sales. See the correct answer for a completeexplanation.

Question 3 - IMA 08-P2-46 - Budget Methodologies

Krouse Company is in the process of developing its operating budget for the coming year. Given below are selecteddata regarding the company's two products, laminated putter heads and forged putter heads, that are sold throughspecialty golf shops.

  Putter Heads

Forged Laminated

Raw materials:

Steel 2 pounds @ $5/pound 1 pound @ $5/pound

Copper None 1 pound @ $15/pound

Direct labor 1/4 hour @ $20/hour 1 hour @ $22/hour

Expected sales (units) 8,200 2,000

Selling price per unit $30 $80

Ending inventory target (units) 100 60Beginning inventory (units) 300 60

Beginning inventory (cost) $5,250 $3,120

Manufacturing overhead is applied to units produced on the basis of direct labor hours. Variable manufacturingoverhead is projected to be $25,000, and fixed manufacturing overhead is expected to be $15,000.

The estimated cost to produce one unit of the laminated putter head is

 A. $52.B. 42.C. $46.D. $62.

 A.

We need to (1) determine the overhead application rate to be used, (2) calculate the amount of overhead tobe applied to each unit of laminated putter heads, and then (3) use that along with the other informationgiven on direct materials and direct labor cost to calculate the total cost for one laminated putter head.

(1) Determine the overhead application rate: We will use a comb ined overhead application rate (variable andfixed OH), since the problem does not give enough information to split it out . Since the overhead is to beapplied to both products, we must have production amounts for both products in order to determine theapplication rate per hour of direct labor to be used in producing both produc ts.

(1a) Calculate the number of units to be manufactured of each product: The inventory equation is Beginning

Inventory in Units + Manufactured Units

 Sold Units = Ending Inventory in Units

Forged: 300 + Manufactured Units  8,200 = 100. Manufactured Forged Uni ts = 8,000.

Part 1 : 07/28/10 08:28:23

(c) HOCK international, page 2

Page 3: Budget Methodologies

8/20/2019 Budget Methodologies

http://slidepdf.com/reader/full/budget-methodologies 3/61

Laminated: 60 + Manufactured Units  2,000 = 60. Manufactured Laminated Unit s = 2,000.

(1b) Calculate the number of direct labor hours required for production of both products:

Forged: 8,000 units × 1/4 hour per unit = 2,000 hours.

Laminated: 2,000 units × 1 hour per un it = 2,000 hours.

Total number of d irect labor hours: 2,000 + 2,000 = 4,000.

(2) Calculate the amount of overhead to be applied to each unit of Laminated:

Total overhead of $25,000 variable plus $15,000 fixed = $40,000. $40,000 divided by the 4,000 total number ofdirect labor hours = $10 per direct labor hour.

Each uni t of Laminated requires 1 hour of d irect labor. Therefore, the amount of overhead to be applied toeach unit of Laminated w ill be $10 × 1, or $10.

(3) Calculate the total cost fo r one Laminated Putter Head:

Raw materials:1 pound steel @ $5/pound $ 5

1 pound copper @ $15/pound 15

Direct labor:

1 hour @ $22/hour 22

Overhead:

1 direc t labor hour @ $10/DLH 10

Total $52

B. This is the material and labor cost for a laminated putter, but it does not include any applied overhead.

C. This amount includes applied overhead in the amount of $4 per unit, which is total overhead of $40,000 divided bythe 10,000 total units to be produced. However, the problem indicates that overhead should be based on direct laborhours used, and the two products do not require the same number of direct labor hours to manufacture. The numberof hours required to produce each unit provides the basis for overhead allocation.

D. This amount includes applied overhead in the amount of $20, which is total overhead of $40,000 divided by the2,000 total units to be produced of Laminated Putter Heads. This means that all of the overhead is being applied tothe Laminated Putter Heads and none to the Forged Putter Heads. The total overhead needs to be allocated to bothproducts on the basis of direct labor hours used by each product.

Question 4 - CMA 692 3-15 - Budget Methodologies

 An organization that specializes in reviewing and editing technical magazine articles sets the following standards forevaluating the performance of the professional staff:

 Annual budgeted fixed costs for normal capacity level of 10,000 articles reviewed and edited: $600,000

Standard professional hours per 10 articles: 200 hours

Flexible budget of standard labor costs to process 10,000 articles: $10,000,000

The following data apply to the 9,500 articles that were actually reviewed and edited during the current year:

Total hours used by professional staff: 192,000 hours

Flexible costs: $9,120,000

Total cost: 9,738,000

Part 1 : 07/28/10 08:28:23

(c) HOCK international, page 3

Page 4: Budget Methodologies

8/20/2019 Budget Methodologies

http://slidepdf.com/reader/full/budget-methodologies 4/61

Using a flexible budget, the total cost planned for the review and editing of 9,500 articles should be

 A. $10,570,000.B. $9,500,000.C. $10,100,000.D. $10,070,000.

 A. Fixed costs are the same ($600,000) regardless to the level of production. This answer is calculated incorrectly byadjusting the fixed costs downward for production. Variable labor costs, however, change with the level of productionand need to be $9,500,000 [($10,000,000 ÷ 10,000) × 9,500].

B. This is the variable labor costs only [($10,000,000 / 10,000) × 9,500].

C. The annual budgeted fixed cos ts for the normal capacity level of 10,000 articles reviewed and edited is$600,000 and it is the same for any other level of output, as fixed costs do no t vary with the production level.The standard labor costs for the 9,500 articles is $9,500,000 [($10,000,000 ÷ 10,000) × 9,500]. The fixed costsplus the labor cost equal the total flexible budget cost planned for the review and editing of 9,500 articles:$600,000 + $9,500,000 = $10,100,000.

D. Fixed costs are the same regardless to the level of production. This answer is calculated incorrectly by adjusting

the fixed costs downward for production.

Question 5 - IMA 08-P2-08 - Budget Methodologies

Which one of the following is not an advantage of a participatory budgeting process?

 A. Control of uncertainties.B. Coordination between departments.C. Goal congruence.D. Communication between departments.

 A. Under any budgeting process , there w il l always be uncer tai nt ies that cannot be es timated or control led,and this will be true with a participatory budgeting process as much as with any other type of budgetingprocess.

B. This is an advantage of participatory budgeting.

C. This is an advantage of participatory budgeting.

D. This is an advantage of participatory budgeting.

Question 6 - CMA 1294 3-7 - Budget Methodologies

Super Drive, a computer disk storage and back-up company, uses accrual accounting. The company's Statement ofFinancial Position for the year ended November 30, is as fol lows:

Super DriveStatement of Financial Position

November 30

 Assets

Cash $52,000

 Accounts receivable, net. 150,000

Inventory 315,000Property, plant and equipment 1,000,000

Total assets $1,517,000

Part 1 : 07/28/10 08:28:23

(c) HOCK international, page 4

Page 5: Budget Methodologies

8/20/2019 Budget Methodologies

http://slidepdf.com/reader/full/budget-methodologies 5/61

Liabilities and Equity 

 Accounts payable $175,000

Common stock 900,000

Retained earnings 442,000

Total liabilities and shareholders equity $1,517,000

 Additional information regarding Super Drive's operations include the following:Sales are budgeted at $520,000 for December and $500,000 for January of the next year.

Collections are expected to be 60% in the month of sale and 40% in the month following the sale.

80% of the disk drive components are purchased in the month prior to the month of sale, and 20% arepurchased in the month of sale. Purchased components are 40% of the cost of goods sold.

Payment for the components is made in the month following the purchase.

Cost of goods sold is 80% of sales.

The budgeted cash collections for the month of December are

 A. $462,000

B. $520,000C. $402,000D. $208,000

 A. Cash co llect ions are equal to 60% of the December sales plus all o f the accounts receivab le from thebeginning of the period: ($520,000 × 60%) + $150,000 = $462,000.

B. This is the budgeted level of sales for December. See the correct answer for a complete explanation.

C. This answer is incorrect. See the correct answer for a complete explanation.

D. This is 40% of the budgeted December sales. See the correct answer for a complete explanation.

Question 7 - IMA 08-P2-36 - Budget Methodologies

Netco's sales budget for the coming year is as follows.

Item Volume in UnitsSales Prices Sales Revenue

1 200,000 $50 $10,000,000

2 150,000 $10 1,500,000

3 300,000 $30 9,000,000

  Total Sales Revenue: $20,500,000

Items 1 and 3 are different models of the same product. Item 2 is a complement to Item 1. Past experience indicatesthat the sales volume of Item 2 relative to the sales volume of Item 1 is fairly constant. Netco is considering a 10%price increase for the coming year for Item 1, which will cause sales of Item 1 to decline by 20%, whilesimultaneously causing sales of Item 3 to increase by 5%. If Netco institutes the price increase for Item 1, total salesrevenue will decrease by

 A. $850,000B. $1,050,000C. $550,000D. $750,000

 A. This answer results from reducing Product 1's quantity by 10% instead of 20% and from not changing the price forProduct 1.

B.

Part 1 : 07/28/10 08:28:23

(c) HOCK international, page 5

Page 6: Budget Methodologies

8/20/2019 Budget Methodologies

http://slidepdf.com/reader/full/budget-methodologies 6/61

To solve this, work out a revised budgeted revenue figure, compare it with the original budgeted revenue asgiven in the prob lem, and the difference is the answer, as follows:

Product Quantity Price Revenue

1 (200,000 * .80) ($50 * 1.1) $ 8,800,000

2 (150,000 * .80) $10 1,200,000

3 (300,000 * 1.05) $30 9,450,000

  $19,450,000  Original Revenue: 20,500,000

  Decrease: $ 1,050,000

C. This answer is incorrect. See the correct answer for an explanation.

D. This answer results from not calculating the reduced sales of Product 2. The problem states that Product 2 is acomplement of Product 1, and that the sales volume of Item 2 relative to the sales volume of Item 1 is fairly constant.Therefore, if the quantity sold of Product 1 decreases, the quantity sold of Product 2 will decrease by aproportionately equal amount.

Question 8 - CMA 1294 3-18 - Budget Methodologies

Simpson Inc. is in the process of preparing its annual budget. The following beginning and ending inventory levels (inunits) are planned for the year ending December 31.

 BeginningInventory

EndingInventory

Raw material* 40,000 50,000

Work-in-process 10,000 10,000

Finished goods 80,000 50,000

*Two units of raw material are needed to produce each unit of finished product.

If 500,000 finished units were to be manufactured for the year by Simpson Inc., the units of raw material that must bepurchased would be

 A. 990,000 units.B. 1,000,000 units.C. 1,020,000 units.D. 1,010,000 units.

 A. This answer incorporates a decrease in the raw materials inventory during the year instead of an increase. Seethe correct answer for a complete explanation.

B. This is the number of units of raw materials needed for production. However, it does not take the materialsinventory level increase into consideration. See the correct answer for a complete explanation.

C. This answer is incorrect. See the correct answer for a complete explanation.

D. We know that two units o f material are needed to produce each unit of finished good. Since 500,000finished units are to be manufactured, the number of unit s of raw materials needed for production is1,000,000. The inventor y level o f raw materials also is to increase during the year by 10,000, so 1,010,000units o f raw materials will need to be purchased during the year.

Question 9 - CMA 1295 H1 - Budget Methodologies

Which one of the following statements regarding the difference between a flexible budget and a static budget is

Part 1 : 07/28/10 08:28:23

(c) HOCK international, page 6

Page 7: Budget Methodologies

8/20/2019 Budget Methodologies

http://slidepdf.com/reader/full/budget-methodologies 7/61

correct?

 A. A flexible budget is established by operating management, while a static budget is determined by topmanagement.B. A flexible budget primarily is prepared for planning purposes, while a static budget is prepared for performanceevaluation.C. A flexible budget includes only variable costs, whereas a static budget includes only fixed costs.D. A flexible budget provides cost allowances for different levels of activity, whereas a static budget provides costsfor one level of activity.

 A. There are a number of methods of developing budgets: participative budgeting, bottom-up budgeting, top-downapproach, etc. Any level of management can establish either a flexible budget or a static budget.

B. Both, flexible and static budget are usually prepared for planning and performance evaluation purposes.

C. Both flexible and static budget include variable and fixed costs.

D. The static budget is based on the level of output planned at the start of the budget period. A flexiblebudget is developed using budgeted revenues and costs amounts based on the level of actual outputachieved in the budget period. The major difference between a flexible budget and a static budget is the use

of the actual output level in the flexible budget, whereas the static budget uses the output level planned atthe beginning of the budget period.

Question 10 - CMA 1289 4-8 - Budget Methodologies

The foundation of a prof it plan is the

 A. Capital budget.B. Sales forecast.C. Cost and expense budget.

D. Production plan.

 A. The capital budget is the budget in which all capital (property, plant and equipment) expenditures are planned.This budget is not directly connected to all of the current period budgets and it is often prepared years in advance sothat the company is able to obtain the necessary financing or accumulate the necessary cash to carry out its capitalexpansion plans.

B. The sales forecast is usually the first fo recast to be prepared in the budgeting process. After the saleslevel has been determined, the production budget and expense budget are prepared. Only after all of theseother budgets are done can profit s be estimated.

C. Cost and expense budgets can not be calculated until the sales level is estimated.

D. The production plan can not be calculated until the sales level is estimated.

Question 11 - CMA 1293 H2 - Budget Methodologies

The Raymar Company is preparing its cash budget for the months of April and May. The firm has established a$200,000 line of credit with its bank at a 12% annual rate of interest on which borrowings for cash deficits must bemade in $10,000 increments. There is no outstanding balance on the line of credit loan on April 1. Principalrepayments are to be made in any month in which there is a surplus of cash. Interest is to be paid monthly. If thereare no outstanding balances on the loans, Raymar will invest any cash in excess of its desired end-of-month cashbalance in U.S. Treasury bills. Raymar intends to maintain a minimum balance of $100,000 at the end of each

month by either borrowing for deficits below the minimum balance or investing any excess cash. Monthly collectionand disbursement patterns are expected to be:

Collections. 50% of the current month's sales budget and 50% of the previous month's sales budget.

Part 1 : 07/28/10 08:28:23

(c) HOCK international, page 7

Page 8: Budget Methodologies

8/20/2019 Budget Methodologies

http://slidepdf.com/reader/full/budget-methodologies 8/61

 Accounts Payable Disbursements. 75% of the current month's accounts payable budget and 25% of theprevious month's accounts payable budget.

 All other disbursements occur in the month in which they are budgeted.

Budget Information

  March April MaySales $40,000 $50,000 $100,000

 Accounts payable 30,000 40,000 40,000

Payroll 60,000 70,000 50,000

Other disbursements 25,000 30,000 10,000

In May, Raymar's budget will result in

 A. Repay $20,000 principal and pay $1,000 interest.B. Pay $900 interest.C. Borrow an additional $20,000 and pay $1,000 interest.D. Repay $90,000 principal and pay $100 interest.

 A. Principal repayments are to be made in any month in which there is a surplus of cash. There was no cash surplusin May so no principal would be repaid. See the correct answer for a complete explanation.

B. Interest was $1,000, not $900. Additional borrowings are needed as there is a cash deficit in May. See the correctanswer for a complete explanation.

C.

First, we need to determine what the interest is that has to be paid in May because interest is paid monthly.To do this we need to determine if there were any borrow ings made in April.

To determine what the April borrow ings were, we need to first determine the cash co llections for April: 50%of April sales and 50% of March sales (or $25,000 + $20,000 = $45,000) wi ll be co llected in Apr il. Then, weneed to determine the amount paid for account s payable in April: 75% of April AP and 25% of March AP (or$30,000 + $7,500 = $37,500) will be paid in April. Other di sbursements are paid in the month they occur, andfor April they are: $70,000 for payro ll plus $30,000 for o ther disbursements , totaling $100,000.

Subtracting the amount of cash outf lows fr om cash inflows we get a $92,500 negative cash flow for themonth . We assume that beginn ing cash was $100,000 and ending cash also needed to be $100,000.Therefore, the cash deficit for the month was the same as the cash flow: ($92,500).

Since borrowings fo r cash deficit s must be made in $10,000 increments, the company needed to bor row$100,000 to cover the $92,500 cash deficit so it could end the month w ith $100,000 in cash. Thus, we need topay $1,000 of interest on May 1 ($100,000 × (12% ÷ 12)). Note that we have a smal l cash surplus of $7,500 atthe beginning of May since we borrowed more than needed to cover deficit ($100,000  $92,500). Thebeginning cash balance is therefore $100,000 + $7,500, or $107,500.

Now we need to determine cash inflows and outf lows in May as we did for April. Cash co llections in May are50% of the Apr il and May sales (50% × $50,000 + 50% × $100,000 = $75,000). Accounts payable that will bepaid in May are 75% of May's AP and 25% of April's AP ($40,000 × 75% + $40,000 × 25% = $40,000). Otherdisbursements total $61,000 ($50,000 for payroll + $10,000 in other disbu rsements + $1,000 in interest forborrowings during April). Subtracting the total disbursements from the collections in May we get a $26,000negative cash flow ($75,000  $40,000  $61,000). However, adding to th is ($25,000) to the cash surp lus f romthe beginning o f the month w e will get a deficit of for the month of only $18,500. Since borrow ings for cashdeficits must be made in $10,000 increments, the company needs to borrow $20,000 to cover the $18,500cash deficit of May and end the month with at least $100,000 in cash.

D. Principal repayments are to be made in any month in which there is a surplus of cash. There was no cash surplusin May so no principal would be repaid. Also the interest is not $100. See the correct answer for a complete

explanation.

Part 1 : 07/28/10 08:28:23

(c) HOCK international, page 8

Page 9: Budget Methodologies

8/20/2019 Budget Methodologies

http://slidepdf.com/reader/full/budget-methodologies 9/61

Question 12 - CIA 590 IV-12 - Budget Methodologies

 A firm desires a finished goods ending inventory equal to 25% of the following month's budgeted sales. Januarysales are budgeted at 10,000 units and February at 12,000 units. Each unit requires 2 pounds of Material X, whichcosts $4 per pound. The company has a just-in-time system and materials are delivered daily just prior to use, so no

raw materials inventories are maintained. Materials are paid for in the month fol lowing purchase. The January 1finished goods inventory is 2,500 units. In February, what amount should the company expect to pay as a cashoutflow for raw materials?

 A. $21,000B. $84,000C. $40,000D. $42,000

 A. This is the number of raw material purchases in January in pounds. See the correct answer for a completeexplanation.

B.

Since no materials inventory is kept on hand, the amount of materials purchased each month is equal to theproduction requirements. Thus, the first thing we need to determine is the finished goods production inJanuary by using formula of physical flow of goods: Beginning Inventory + Units Produced  Units Sold =Ending Inventory.

Plugging the numbers for fin ished goods into the formula we get: 2,500 + Units Produced  10,000 = (.25 ×12,000), or 2,500 + Units Produced  10,000 = 3,000. Solving for Units Produced, we get Units Produced =10,500.

Since the company makes payment the month after the purchase, January raw material purchases will bepaid in February. Now we can determine the cash outlay for raw materials in February: 10,500 unit s × 2 lb. *$4.00 = $84,000.

C. This answer is incorrect. See the correct answer for a complete explanation.

D. This answer ignores the fact that 2 lb. is necessary to produce one unit of f inished product. See the correctanswer for a complete explanation.

Question 13 - CMA 1296 H9 - Budget Methodologies

Karmee Company has been accumulating operating data in order to prepare an annual profit plan. Details regardingKarmee's sales for the first 6 months of the coming year are as follows:

MonthEstimated

Monthly Sales Type of 

Monthly Sale 

January $600,000

 All Months:

Cash sales 20%Credit sales 80%

February 650,000

March 700,000

 April 625,000

May 720,000

June 800,000

Collection Pattern for Credit Sales

Month of sale 30%One month following sale 40%

Part 1 : 07/28/10 08:28:23

(c) HOCK international, page 9

Page 10: Budget Methodologies

8/20/2019 Budget Methodologies

http://slidepdf.com/reader/full/budget-methodologies 10/61

Second month following sale 25%

Karmee's cost of goods sold averages 40% of the sales value. Karmee's objective is to maintain a target inventoryequal to 30% of the next month's sales in units. Purchases of merchandise for resale are paid for in the monthfollowing the sale. The variable operating expenses (other than cost of goods sold) for Karmee are 10% of sales andare paid for in the month following the sale. The annual fixed operating expenses are presented below. All of theseare incurred uniformly throughout the year and paid monthly except for insurance and property taxes. Insurance ispaid quarterly in January, April, July, and October. Property taxes are paid twice a year in April and October.

 Annual Fixed Operating Costs

 Advertising $720,000

Depreciation 420,000

Insurance 180,000

Property taxes 240,000

Salaries 1,080,000

Karmee Company's total cash receipts for the month of April will be

 A. $707,400B. $504,000C. $653,000D. $629,000

 A. This is the collections in the month of June. See the correct answer for a complete explanation.

B. This answer does not include the cash sales in April. See the correct answer for a complete explanation.

C. This answer includes the bad debts from January sales. See the correct answer for a complete explanation.

D. In April the company receives the amount o f April cash sales and the cash remitted for credit sales. Weknow that 80% of sales are credit sales. In April the company w ill receive 30% of April c redit sales, 40% of

March credit sales, and 25% of February credit sales. The cash sales in April wi ll be $125,000 (20% ×$625,000). The cash collections in April from credit sales w ill be as follows: From April credit sales cashcollect ions will be $150,000 ($625,000 × 80% × 30%). From March credit sales cash collections will be$224,000 ($700,000 × 80% × 40%). From February credit sales cash collec tions will be $130,000 ($650,000 ×80% × 25%). Adding these amount s together, the total cash collections in April will be $629,000 ($125,000 +$150,000 + $224,000 + $130,000).

Question 14 - CMA 1296 H10 - Budget Methodologies

Karmee Company has been accumulating operating data in order to prepare an annual profit plan. Details regarding

Karmee's sales for the first 6 months of the coming year are as follows:

MonthEstimated

Monthly Sales Type of 

Monthly Sale 

January $600,000

 All Months:

Cash sales 20%Credit sales 80%

February 650,000

March 700,000

 April 625,000

May 720,000

June 800,000

Collection Pattern for Credit Sales

Month of sale 30%

Part 1 : 07/28/10 08:28:23

(c) HOCK international, page 10

Page 11: Budget Methodologies

8/20/2019 Budget Methodologies

http://slidepdf.com/reader/full/budget-methodologies 11/61

One month following sale 40%

Second month following sale 25%

Karmee's cost of goods sold averages 40% of the sales value. Karmee's objective is to maintain a target inventoryequal to 30% of the next month's sales in units. Purchases of merchandise for resale are paid for in the monthfollowing the sale. The variable operating expenses (other than cost of goods sold) for Karmee are 10% of sales and

are paid for in the month following the sale. The annual fixed operating expenses are presented below. All of theseare incurred uniformly throughout the year and paid monthly except for insurance and property taxes. Insurance ispaid quarterly in January, April, July, and October. Property taxes are paid twice a year in April and October.

 Annual Fixed Operating Costs

 Advertising $720,000

Depreciation 420,000

Insurance 180,000

Property taxes 240,000

Salaries 1,080,000

The amount of cash collected in March for Karmee Company from the sales made during March will be

 A. $168,000B. $140,000C. $636,000D. $308,000

 A. This is the amount of cash collected in March from credit sales made in March. However, the amount of cashcollected from cash sales made in March must be included as well.

B. This is the amount of cash collected in March from cash sales made in March. However, the amount of cashcollected from credit sales made in March must be included as well.

C. This is the total cash collections in March and it includes collections from previous months' sales as well as fromMarch sales. However, the question asks for only the amount of cash collected in March from the sales madeduring March.

D. In March Karmee Company wi ll collect $140,000 (20% × $700,000) from the cash sales made in March and$168,000 ($700,000 × 80% × 30%) from the credit sales made in March. The total amount of March salescollected in March is $308,000.

Question 15 - CMA 1296 H5 - Budget Methodologies

The budgeting process should be one that motivates managers and employees to work toward organizational goals.

Which one of the following is least likely to motivate managers?

 A. Participation by subordinates in the budgetary process.B. Holding subordinates accountable for the items they control.C. Use of management by exception.D. Having top management set budget levels.

 A. Participation in the budgetary process would motivate managers by giving them a sense of ownership of thebudget.

B. Holding subordinates accountable for the items they control would most likely motivate managers and encouragea better performance.

C. Management by exception lets managers concentrate their attention on areas where problems are. This will l ikelybe a motivating item for managers as they will feel that their efforts are being directed where they are needed.

Part 1 : 07/28/10 08:28:23

(c) HOCK international, page 11

Page 12: Budget Methodologies

8/20/2019 Budget Methodologies

http://slidepdf.com/reader/full/budget-methodologies 12/61

D. A budget is a very useful tool in a number of areas: planning , control, evaluation, motivation,communication, identifying future problems. However, when top management sets the budget w ithout anyparticipation f rom those who will be responsib le for meeting the budgeted goals, employees will not have asense of ownership of the plan. This will not be motivating and thus, not very effective.

Question 16 - CMA 691 3-15 - Budget Methodologies

Which one of the following schedules would be the last item to be prepared in the normal budget preparationprocess?

 A. Cash budget.B. Direct labor budget.C. Manufacturing overhead budget.D. Cost of goods sold budget.

 A. The cash budget is the las t budget to be prepared because all other budgets are inputs to it .

B. The direct labor budget is part of the production budget, which is part of the operating budget. The productionbudget is developed after the sales budget, but it is not the last budget to be prepared.

C. The manufacturing overhead budget is part of the production budget, which is part of the operating budget. Theproduction budget is developed after the sales budget, but it is not the last budget to be prepared.

D. The cost of goods sold budget is produced after the production budget. It is a part of the operating budget but it isnot the last budget to be prepared.

Question 17 - CMA 1291 3-25 - Budget Methodologies

Information pertaining to Noskey Corporation's sales revenue is presented in the following table.

 November 

Year 1(Actual)

December Year 1

(Budget)

JanuaryYear 2

(Budget)

Cash sales $80,000 $100,000 $60,000

Credit sales 240,000 360,000 180,000

Total sales $320,000 $460,000 $240,000

Management estimates that 5% of credit sales are uncollectible. Of the credit sales that are collectible, 60% arecollected in the month of sale and the remainder in the month fol lowing the sale. Purchases of inventory are equal to

next month's sales and gross profit margin is 30%. All purchases of inventory are on account; 25% are paid in themonth of purchase, and the remainder are paid in the month following the purchase.

Noskey Corporation's budgeted total cash payments in December Year 1 for inventory purchases are

 A. $168,000B. $405,000C. $283,500D. $220,500

 A. This is the amount of purchases in December (of which only 25% are actually paid in December). See the correctanswer for a complete explanation.

B. This is purchases valued at the sales price, not the cost of goods sold (70% of purchased price). See the correctanswer for a complete explanation.

Part 1 : 07/28/10 08:28:23

(c) HOCK international, page 12

Page 13: Budget Methodologies

8/20/2019 Budget Methodologies

http://slidepdf.com/reader/full/budget-methodologies 13/61

C. In December the cash payments w ill be for 25% of the purchases made in December and for 75% of thepurchase made in November. December purchases are for the budgeted January sales and are equal to$168,000 ($240,000 total January sales × 70% COGS). Thus , 25% of the December purchases is $42,000($168,000 × 25%). November pu rchases were for the December sales and are equal to $322,000 ($460,000total December sales × 70% COGS). Thus, 75% of November purchases is $241,500 ($322,000 × 75%). Addingthese two cash payments together, we get a total cash payment in December of $283,500 ($42,000 +$241,500).

D. This answer is calculated based on credit sales only, and not total sales. See the correct answer for a completeexplanation.

Question 18 - CMA 691 3-12 - Budget Methodologies

Flexible budgets

 A. Accommodate changes in the inflation rate.B. Are used to evaluate capacity use.

C. Accommodate changes in activity levels.D. Are static budgets that have been revised for changes in prices.

 A. The inflation rate may be considered in various types of budgets, for example a fixed or static budget, a projectbudget, or a flexible budget.

B. Flexible budgets are prepared for different levels of activity and do not evaluate the usage of capacity.

C. Flexible budgets are prepared for different levels of activit y. In other words , a flexible budget is based onthe static budget, but variable revenues and variable expenses are adjusted upward or downw ard to reflectthe actual activity level. A flexible budget is more useful than a static budgetsas it allows the company tocompare the actual results with the budgeted results that have been adjusted for the actual level of activity.

D. Flexible budgets are prepared for different levels of activity. In other words, a flexible budget is based on the staticbudget, but variable revenues and variable expenses are adjusted upward or downward to reflect the actual activitylevel. The actual activity level might well change when prices change, but the flexible budget does not incorporatechanges in prices in its budgeted amounts, only changes in volume.

Question 19 - CMA 1296 H7 - Budget Methodologies

Karmee Company has been accumulating operating data in order to prepare an annual profit plan. Details regardingKarmee's sales for the first 6 months of the coming year are as follows:

Month EstimatedMonthly Sales 

Type of Monthly Sale 

January $600,000

 All Months:

Cash sales 20%Credit sales 80%

February 650,000

March 700,000

 April 625,000

May 720,000

June 800,000

Collection Pattern for Credit Sales

Month of sale 30%

One month following sale 40%

Second month following sale 25%

Part 1 : 07/28/10 08:28:23

(c) HOCK international, page 13

Page 14: Budget Methodologies

8/20/2019 Budget Methodologies

http://slidepdf.com/reader/full/budget-methodologies 14/61

Karmee's cost of goods sold averages 40% of the sales value. Karmee's objective is to maintain a target inventoryequal to 30% of the next month's sales in units. Purchases of merchandise for resale are paid for in the monthfollowing the sale. The variable operating expenses (other than cost of goods sold) for Karmee are 10% of sales andare paid for in the month following the sale. The annual fixed operating expenses are presented below. All of theseare incurred uniformly throughout the year and paid monthly except for insurance and property taxes. Insurance ispaid quarterly in January, April, July, and October. Property taxes are paid twice a year in April and October.

 Annual Fixed Operating Costs

 Advertising $720,000

Depreciation 420,000

Insurance 180,000

Property taxes 240,000

Salaries 1,080,000

The amount for cost of goods sold that will appear on Karmee Company's budgeted income statement for themonth of February will be

 A. $254,000B. $195,000C. $260,000D. $272,000

 A. This answer is incorrect. See the correct answer for a complete explanation.

B. This answer is incorrect. See the correct answer for a complete explanation.

C. The sales are estimated to be $650,000 in February and t he cost of goods sold averages 40% of the salesvalue. Thus , the cos t of goods sold in February is $260,000 ($650,000 × 40%).

D. This answer is incorrect. See the correct answer for a complete explanation.

Question 20 - CMA 1291 3-23 - Budget Methodologies

Information pertaining to Noskey Corporation's sales revenue is presented in the following table.

 November 

Year 1(Actual)

December Year 1

(Budget)

JanuaryYear 2

(Budget)

Cash sales $80,000 $100,000 $60,000

Credit sales 240,000 360,000 180,000

Total sales $320,000 $460,000 $240,000

Management estimates that 5% of credit sales are uncollectible. Of the credit sales that are collectible, 60% arecollected in the month of sale and the remainder in the month fol lowing the sale. Purchases of inventory are equal tonext month's sales and gross profit margin is 30%. All purchases of inventory are on account; 25% are paid in themonth of purchase, and the remainder are paid in the month following the purchase.

Noskey Corporation's budgeted cash collections in December Year 1 from November Year 1 credit sales are

 A. $91,200B. $228,000C. $136,800D. $84,000

 A. November c redit sales are $240,000. However, 5% of th is to tal is no t col lectible. Therefo re, the totalNovember c redit sales that w ill be collectib le are only $228,000 ($240,000 × 95%). Of this amount, 40% of the

Part 1 : 07/28/10 08:28:23

(c) HOCK international, page 14

Page 15: Budget Methodologies

8/20/2019 Budget Methodologies

http://slidepdf.com/reader/full/budget-methodologies 15/61

collectible November credit sales will be collected in December. This is $91,200 ($228,000 × 40%).

B. This is the estimated credit sales made in November minus the uncollectible amount of November credit sales.See the correct answer for a complete explanation.

C. This is the cash collection of November sales in November. See the correct answer for a complete explanation.

D. This represents 35% of the November credit sales. See the correct answer for a complete explanation.

Question 21 - CMA 691 3-4 - Budget Methodologies

DeBerg Company has developed the following sales projections for the calendar year.

May $100,000

June 120,000

July 140,000

 August 160,000September 150,000

October 130,000

Normal cash collection experience has been that 50% of sales are collected during the month of sale and 45% inthe month fol lowing sale. The remaining 5% of sales is never collected. DeBerg's budgeted cash collections for thethird calendar quarter are

 A. $422,500B. $414,000C. $427,500D. $450,000

 A. This is the budgeted collections for the periods from August through October. See the correct answer for acomplete explanation.

B.

The third calendar quarter is represented by July, August and September. In each of these months DeBerg'scollects 50% of that month's sales plus 45% of the p revious month's sales as follows :

Ju ly: ($140,000 × 50%) + ($120,00 × 45%) = $124,000

 August: ($160,000 × 50%) + ($140,000 × 45%) = $143,000

September: ($150,000 * 50%) + ($160,000 × 45%) = $147,000

In to tal, co llect ions will be $124,000 + $143,000 + $147,000 = $414,000.

This question can also be so lved by assuming that in the third quarter, the company collect 95% of July and August sales plus 45% of June sales and 50% of September sales.

C. This is the amount of cash collected from sales made in the third calendar quarter.

D. This is the total sales for the third calendar quarter, not the cash collected.

Question 22 - CMA 1291 H2 - Budget Methodologies

Part 1 : 07/28/10 08:28:23

(c) HOCK international, page 15

Page 16: Budget Methodologies

8/20/2019 Budget Methodologies

http://slidepdf.com/reader/full/budget-methodologies 16/61

 A planning calendar in budgeting is the

 A. Schedule of activities for the development and adoption of the budget.B. Sales forecast by months in the annual budget period.C. Calendar period covered by the annual budget and the long-range plan.D. Calendar period covered by the budget.

 A. A planning calendar is a document that sets for th all of the deadl ines as well as the pol icies andprocedures for the budgeting process. Therefore, it includes the scheduled activities and deadlines for thebudgeting process.

B. A planning calendar is not the monthly sales forecast. A planning calendar is a document that sets forth all of thedeadlines as well as the policies and procedures for the budgeting process.

C. A planning calendar is not the period covered by the annual budget and long-range plan. A planning calendar is adocument that sets forth all of the deadlines as well as the policies and procedures for the budgeting process.

D. A planning calendar is not the period covered by the budget. A planning calendar is a document that sets forth allof the deadlines as well as the policies and procedures for the budgeting process.

Question 23 - CMA 1291 3-20 - Budget Methodologies

 A continuous profit plan

 A. Is a plan that is revised monthly or quarterly.B. Works best for a company that can reliably forecast events a year or more into the future.C. Is an annual plan that is part of a 5-year plan.D. Is a plan devised by a full-time planning staff.

 A. A continuous plan is o ne that is aut omatically prepared for a cer tain period of time ahead o f the present .

For example, a 1-year continuous plan wi ll be prepared at the end of every month for the next 12 months .

B. A continuous plan works for any company, whether or not they can accurately forecast more than a year into thefuture.

C. A continuous plan does not need to be part of a larger 5-year plan.

D. A continuous plan may or may not be produced by a full-time planning staff.

Question 24 - CMA 1291 3-22 - Budget Methodologies

 A systemized approach known as zero-base budgeting (ZBB)

 A. Classifies budget requests by activity and estimates the benefits arising from each activity.B. Presents a statement of expectations for a period of time but does not present a firm commitment.C. Presents the plan for only one level of activity and does not adjust to changes in the level of activity.D. Divides the activities of individual responsibility centers into a series of packages that are prioritized.

 A. Activity-based budgeting, not zero-base budgeting, focuses on the budged cost of activities necessary to produceand sell products and services.

B. Zero-based budgeting represents a firm commitment to the company just like any other budget.

C. This is a description of a static budget, not a zero-base budget. A static budget is a budget that is prepared foronly one level of activity within the company. A zero-base budget may be a static budget, but it may also be a flexiblebudget.

Part 1 : 07/28/10 08:28:23

(c) HOCK international, page 16

Page 17: Budget Methodologies

8/20/2019 Budget Methodologies

http://slidepdf.com/reader/full/budget-methodologies 17/61

D.

Zero-based budgeting is the budgeting method in which the current year budget is prepared without anyreference to, or use of, the prior period's budget. This is quite different from a budget that is prepared bylooking at the current year's actual or budgeted amounts and simply adjusting them (usually increasingthem) for expected changes in the coming year.

Though th is method is more time consuming and diffi cult for all of the people involved, there are a numberof advantages to the company as a result of using th is method. Because the budget is built up from zero,each manager must justify all of the expenses in his or her department. Each component is evaluated from acost -benefit perspective and priorities are determined. Zero-based budgeting enables the company toidentify expenses that are not value adding or that should be reduced due to some development inproduction methods or something similar.

Question 25 - CMA 1295 H2 - Budget Methodologies

Based on past experience, a company has developed the following budget formula for estimating its shippingexpenses. The company's shipments average 12 lbs. per shipment:

Shipping costs = $16,000 + ($0.50 x lbs. shipped)

The planned activity and actual activity regarding orders and shipments for the current month are given in thefollowing schedule:

  Plan Actual

Sales orders 800 780

Shipments 800 820

Units shipped 8,000 9,000

Sales $120,000 $144,000Total pounds shipped 9,600 12,300

The actual shipping costs for the month amounted to $21,000. The appropriate monthly flexible budget allowancefor shipping costs for the purpose of performance evaluation would be

 A. $22,150B. $20,800C. $20,920D. $20,680

 A. There is a lot of informat ion in this ques tion, but much of i t is not needed. We are told what the formula isand that the actual sh ipping was 12,300 pounds. Putting t his in to the formula, we get: $16,000 + ($.50 ×

12,300) = $22,150.

B. This answer uses the planned number of pounds to be shipped. See the correct answer for a completeexplanation.

C. This answer uses the number of shipments instead of the number of pounds shipped. See the correct answer fora complete explanation.

D. This answer uses the number of orders instead of the number of pounds shipped. See the correct answer for acomplete explanation.

Question 26 - CMA 1283 4-24 - Budget Methodologies

Part 1 : 07/28/10 08:28:23

(c) HOCK international, page 17

Page 18: Budget Methodologies

8/20/2019 Budget Methodologies

http://slidepdf.com/reader/full/budget-methodologies 18/61

Kelly Company is a retail sporting goods store that uses accrual accounting for its records. Facts regarding Kelly'soperations are as follows:

Sales are budgeted at $220,000 for December year 1 and $200,000 for January year 2.

Collections are expected to be 60% in the month of sale and 38% in the month following the sale.

Gross margin is 25% of sales.

 A total of 80% of the merchandise held for resale is purchased in the month prior to the month of sale and20% is purchased in the month of sale. Payment for merchandise is made in the month following thepurchase.

Other expected monthly expenses to be paid in cash are $22,600.

 Annual depreciation is $216,000.

Below is Kelly Company's statement of financial position at November 30, year 1.

 Assets  

Cash $ 22,000

 Accounts receivable(net of $4,000 allowance for uncollectible accounts)

76,000

Inventory 132,000Property, plant, and equipment (net of $680,000 accumulated deprecation) 870,000

Total assets $1,100,000

Liabilities and Stockholders' Equity  

 Accounts payable $ 162,000

Common stock 800,000

Retained earnings 138,000

Total liabilities and stockholders' equity $1,100,000

The projected balance in accounts payable on December 31, year 1 is

 A. Some amount other than those given.

B. $162,000.C. $204,000.D. $153,000.

 A. The correct amount is given as one of the other choices.

B. This is AP balance for November. See the correct answer for a complete explanation.

C. This is a level of purchases at sales price. See the correct answer for a complete explanation.

D. The amount of account s payable in December is equal to the December purchases. This is becausepurchases are paid in the month follow ing the purchase. Purchases made in December are made to support20% of December sales and 80% of January sales. Thus, purchases in December are equal to an amount that

wi ll suppor t $204,000 in sales ($220,000 × 20%) + ($200,000 × 80%). However, that $204,000 is the salesamount, which is based on the selling price and not on the company's cost f or the inventory. The company'scost for the inventory is equal to 75% of the selling price, since its gross margin is equal to 25% of theselling price. Therefore, the AP are equal to 75% of estimated sales o f $204,000, or $153,000.

Question 27 - CMA 1294 3-9 - Budget Methodologies

Super Drive, a computer disk storage and back-up company, uses accrual accounting. The company's Statement ofFinancial Position for the year ended November 30, is as fol lows:

Super DriveStatement of Financial PositionNovember 30

Part 1 : 07/28/10 08:28:23

(c) HOCK international, page 18

Page 19: Budget Methodologies

8/20/2019 Budget Methodologies

http://slidepdf.com/reader/full/budget-methodologies 19/61

 Assets

Cash $52,000

 Accounts receivable, net. 150,000

Inventory 315,000

Property, plant and equipment 1,000,000

Total assets $1,517,000

Liabilities and Equity 

 Accounts payable $175,000

Common stock 900,000

Retained earnings 442,000

Total liabilities and shareholders equity $1,517,000

 Additional information regarding Super Drive's operations include the following:

Sales are budgeted at $520,000 for December and $500,000 for January of the next year.

Collections are expected to be 60% in the month of sale and 40% in the month following the sale.

80% of the disk drive components are purchased in the month prior to the month of sale, and 20% arepurchased in the month of sale. Purchased components are 40% of the cost of goods sold.

Payment for the components is made in the month following the purchase.

Cost of goods sold is 80% of sales.

The projected gross profit for the month ending December 31 is

 A. $416,000B. $536,000C. $104,000D. $134,000

 A. This is the cost of goods sold for December. See the correct answer for a complete explanation.

B. Gross profit (gross margin) is equal to sales minus cost of goods sold. Thus, gross profit cannot be greater thanthe amount of sales, and sales are budgeted at $520,000 for December. See the correct answer for a completeexplanation.

C. Gross pro fit (gross margin) is equal to sales minus cos t of goods so ld. December sales are projected tobe $520,000. Cost o f goods sold is 80% of sales. Thus, the projected gross profit is 20% of sales, or $104,000.

D. This answer is incorrect. See the correct answer for a complete explanation.

Question 28 - CIA 594 III-69 - Budget Methodologies

 A company produces a product that requires 2 pounds of a raw material. The company forecasts that there will be6,000 pounds of raw material on hand at the end of June. At the end of any given month the company wishes tohave 30% of next month's raw material requirements on hand. The company has budgeted production of theproduct for July, August, September, and October to be 10,000, 12,000, 13,000, and 11,000 units, respectively. Asof June 1, the raw material sells for $1.00 per pound.

In the month of September, raw material purchases and ending inventory, respectively, will be (in pounds):

 A. 24,800 and 6,600B. 28,600 and 6,600C. 32,600 and 6,600D. 13,000 and 3,900

 A.

We need to determine the purchases of raw materials for September by using t he formula for the physical

Part 1 : 07/28/10 08:28:23

(c) HOCK international, page 19

Page 20: Budget Methodologies

8/20/2019 Budget Methodologies

http://slidepdf.com/reader/full/budget-methodologies 20/61

flow of goods: Beginning inventory + Amount purchased  Production requirements = Ending inventory.

Beginning Inventory in September will be 13,000 × 2 × .30, which equals 7,800 units. Ending Inventory inSeptember w ill be 11,000 × 2 × .30, wh ich equals 6,600 units. Materials used dur ing September will be 13,000× 2, which equals 26,000 unit s. Therefore, the inventory formula is:

7,800 + Purchases  26,000 = 6,600Purchases = 24,800, which is half o f the answer.

The other half of the answer is the Ending Inventory wh ich we calculated above as 6,600 units .

B. The amount of raw material purchases is incorrect. See the correct answer for a complete explanation.

C. The amount of raw material purchases is incorrect. See the correct answer for a complete explanation.

D. 13,000 is the budgeted production in units. 3,900 is the number of finished units that could be produced from theSeptember beginning inventory of 7,800 lb. See the correct answer for a complete explanation.

Question 29 - CMA 1294 H4 - Budget Methodologies

 A continuous (rolling) budget

 A. Presents the plan for a range of activity so that the plan can be adjusted for changes in activity.B. Presents the plan for only one level of activity and does not adjust to changes in the level of activity.C. Drops the current month or quarter and adds a future month or quarter as the current month or quarter iscompleted.D. Presents planned activities for a period but does not present a firm commitment.

 A. A continuous budget can be prepared for only one level of activity. The budget that prepared for different levels ofactivity is called a flexible budget.

B. A continuous budget can be prepared for different levels of activities. The budget that is prepared for only onelevel of activity is a static budget.

C. A continuous budget, also called a rolling budget, is one that is prepared for a certain period of timeahead of the present. For example, a 1-year continuous budget will be prepared at the end of every monthfor the next 12 months.

D. A continuous budget represents the same firm commitment that other types of budgets represent.

Question 30 - CIA 1190 IV-17 - Budget Methodologies

The master budget

 A. Shows forecasted and actual results.B. Reflects controllable costs only.C. Contains the operating budget.D. Can be used to determine manufacturing cost variances.

 A. The master budget is prepared for one level of output and made before or at the start of the budgeting period.The master budget represents what revenues and costs are planned to be for the budgeted period. Master budgetdo not include actual results.

B. The master budget is prepared for one level of output and made before or at the start of the budgeting period.The master budget represents what revenues and costs are planned to be for the budgeted period. It includes allapplicable costs including costs that are not controllable by individual managers.

Part 1 : 07/28/10 08:28:23

(c) HOCK international, page 20

Page 21: Budget Methodologies

8/20/2019 Budget Methodologies

http://slidepdf.com/reader/full/budget-methodologies 21/61

C. The master budget is the composition of pro forma of balance sheet, cash budget, statement of cash flowand the capital budget. Income statement which is the last budget of operating budget which is an input forbalance sheet. Thus, the master budget contains operating budget.

D. The master budget is prepared for one level of output and made before or at the start of the budgeting period.The master budget represents what revenues and costs are planned to be for the budgeted period. To determine

manufacturing cost variances the actual results and flexible budget data have to be used.

Question 31 - CMA 1296 H6 - Budget Methodologies

Karmee Company has been accumulating operating data in order to prepare an annual profit plan. Details regardingKarmee's sales for the first 6 months of the coming year are as follows:

MonthEstimated

Monthly Sales Type of 

Monthly Sale 

January $600,000

 All Months:

Cash sales 20%Credit sales 80%

February 650,000

March 700,000

 April 625,000

May 720,000

June 800,000

Collection Pattern for Credit Sales

Month of sale 30%

One month following sale 40%

Second month following sale 25%

Karmee's cost of goods sold averages 40% of the sales value. Karmee's objective is to maintain a target inventoryequal to 30% of the next month's sales in units. Purchases of merchandise for resale are paid for in the monthfollowing the sale. The variable operating expenses (other than cost of goods sold) for Karmee are 10% of sales andare paid for in the month following the sale. The annual fixed operating expenses are presented below. All of theseare incurred uniformly throughout the year and paid monthly except for insurance and property taxes. Insurance ispaid quarterly in January, April, July, and October. Property taxes are paid twice a year in April and October.

 Annual Fixed Operating Costs

 Advertising $720,000

Depreciation 420,000

Insurance 180,000

Property taxes 240,000Salaries 1,080,000

The total cash disbursements that Karmee Company will make for the operating expenses (expenses other than thecost of goods sold) during the month of April will be

 A. $290,000B. $255,000C. $385,000D. $420,000

 A. This answer does not include variable expenses or advertising and it includes depreciation. See the correctanswer for a complete explanation.

B. This answer does not include variable expenses or advertising. See the correct answer for a complete explanation.

Part 1 : 07/28/10 08:28:23

(c) HOCK international, page 21

Page 22: Budget Methodologies

8/20/2019 Budget Methodologies

http://slidepdf.com/reader/full/budget-methodologies 22/61

C. Operating expenses other that COGS paid in the month of April are: 10% of March's sales, the advertisingmonth ly payment, monthly salaries, the quarterly insurance payment and the property tax payment madetwice a year. Depreciation is not a cash expense and is therefore not included in calculation. Thedisbursements are: $70,000 for variable operating expenses (March sales of $700,000 × 10%), $60,000 foradver tis ing ($720,000 ÷ 12), $90,000 for salaries ($1,080,000 ÷ 12), $45,000 for insurance ($180,000 ÷ 4), and$120,000 for property taxes ($240,000 ÷ 2). The total amount o f d isbursements in April is $385,000.

D. This answer incorrectly includes depreciation. Depreciation is not a cash expenditure. See the correct answer fora complete explanation.

Question 32 - CMA 1289 4-24 - Budget Methodologies

Birch Corporation has the following historical pattern on its credit sales.

70% collected in month of sale15% collected in the first month after sale10% collected in the second month after sale4% collected in the third month after sale1% uncollectible

The sales on open account have been budgeted for the first 6 months of the year are as follows:

Month Sales On

Open Account  

January $70,000

February 90,000

March 100,000

 April 120,000

May 100,000June 90,000

The estimated total cash collections during April from accounts receivable would be

 A. $118,800B. $84,000C. $110,800D. $108,000

 A. This answer is incorrect. See the correct answer for a complete explanation.

B. This is the cash collected from April sales only. See the correct answer for a complete explanation.

C.

Using the cash collection pattern that refers to the credit sales we can estimate how much cash is collectedfrom accounts receivable during April:

$120,000 × 70% = $84,000 (from April c redit sales)$100,000 × 15% = $15,000 (from March c redit sales)$90,000 × 10% = $9,000 (from February credi t sales)$70,000 × 4% = $2,800 (from January cred it sales)

 Add ing al l these numbers together we w il l get $110,800 of cash col lec ted in Apr il f rom credi t sales.

D. This answer is incorrect. See the correct answer for a complete explanation.

Part 1 : 07/28/10 08:28:23

(c) HOCK international, page 22

Page 23: Budget Methodologies

8/20/2019 Budget Methodologies

http://slidepdf.com/reader/full/budget-methodologies 23/61

Question 33 - IMA 08-P2-26 - Budget Methodologies

Country Ovens is a family restaurant chain. Due to an unexpected road construction project, traffic passing by theCountry Ovens restaurant in Newtown has significantly increased. As a result, restaurant volume has similarlyincreased well beyond the level expected. Which type of budget would be most appropriate in helping the restaurant

manager plan for restaurant labor costs?

 A. Rolling budget.B. Flexible budget.C. Activity-based budget.D. Zero-based budget.

 A. A rolling budget is a continual budget that adds one month as another month is completed. This type of budget isa static budget, and a static budget does not make allowance for changes in volume that may take place. Therefore,this would not be the best choice in this situation.

B. This is the most appropriate budget, as it allows the management to measure operating success at anyactivity l evel experienced. This budget can be adjusted upward for increased traffic and, if necessary,

adjusted back down when the construction is completed, providing the owner with accurate budgetvariance information under a variety of circumstances.

C. Activity-based budgeting is similar in concept to activity-based costing. Activities that drive the costs are identified,a budgeted level of activity for each of these drivers is determined based on a budgeted level of production, andbudgeted amounts are developed based on the budgeted level of activity. This is a good basis for developing abudget, but since it depends upon a budgeted level of activity, the budget developed will be a static budget that willnot make allowance for changes in volume that may take place. Therefore, this would not be the best choice in thissituation.

D. Zero-based budgeting is a budgeting method in which the budget is prepared without any reference to, or use of,the current period’s budget and the likely operating results for the current period. Although this approach would beappropriate when the next year's volume is expected to far exceed this year’s sales volume, it is still a static budget.

 And a static budget does not make allowance for changes in volume that may take place. Therefore, this would notbe the best choice in this situation.

Question 34 - CMA 1296 H13 - Budget Methodologies

 An advantage of incremental budgeting when compared with zero-base budgeting is that incremental budgeting

 A. Eliminates functions and duties that have outlived their usefulness.B. Encourages adopting new projects quickly.C. Accepts the existing base as being satisfactory.D. Eliminates the need to review all functions periodically to obtain optimum use of resources.

 A. This is an advantage of zero-based budgeting, not incremental budgeting.

B. There is no difference of new project treatment under both of these budget development approaches.

C. Zero-based budgeting is the budgeting method in which the current year's budget is prepared withoutany reference to, or use of, the prior period's budget or actual amounts. Incremental budgeting assumesthat the previous period's budgeted or actual results are satisfactory, and the budget is calculated byadjusting the previous period budgeted or actual amount by a number, for example 1.1, to allow for c hangesplanned for the new budgeting period. Thus, it is easier to prepare an ncremental budget and lessmanagerial effort is consumed than when the budget is prepared under the ZBB concept.

D. Periodic review of business functions is required regardless the type of budget development approach used.

Part 1 : 07/28/10 08:28:23

(c) HOCK international, page 23

Page 24: Budget Methodologies

8/20/2019 Budget Methodologies

http://slidepdf.com/reader/full/budget-methodologies 24/61

Question 35 - CMA 691 3-9 - Budget Methodologies

In developing a comprehensive budget for a manufacturing company, which one of the following items should bedone first?

 A. Determination of the advertising budget.B. Determination of manufacturing capacity.C. Development of a sales plan.D. Development of the capital budget.

 A. The advertising budget is a part of the operating budget. The operating budget is not the first budget to bedeveloped, because it is derived from other budgets that must be developed first.

B. Determination of manufacturing capacity is a part of the process of developing the sales budget as well as theproduction budget. However, determination of manufacturing capacity is not necessarily the first thing to be done indeveloping a comprehensive budget for a manufacturing company.

C.

The sales plan/forecast is usually prepared first in the budgeting process. The production budget and all theother budgets for the company are derived from the sales budget.

D. The capital budget is usually prepared for periods of several years. Any planned projects affecting the next year'sbudget need to be included in the budget. The capital budget is not developed along with the next year's budget,though, because it is outside of the current year's budget. So it would not be the first budget to be developed.

Question 36 - CMA 1291 3-26 - Budget Methodologies

RedRock Company uses flexible budgeting for cost control. RedRock produced 10,800 units of product during

October, incurring indirect materials costs of $13,000. Its master budget for the year reflected indirect materials costsof $180,000 at a production volume of 144,000 units. A flexible budget for October production would reflect indirectmaterials costs of 

 A. $13,000.B. $13,975.C. $13,500.D. $11,700.

 A. This is the actual indirect materials cost. See the correct answer for a complete explanation.

B. This answer is incorrect. See the correct answer for a complete explanation.

C. Indirect materials are variable costs . Therefore, the total cost o f indirect materials fluctuates with the levelof production. According to the master budget, the per unit cost of indirect material is $1.25 ($180,000 ÷144,000). Therefore, given an actual production of 10,800 units, the flexible budget costs for indirect materialin October would be $13,500 ($1.25 × 10,800).

D. This answer is incorrect. See the correct answer for a complete explanation.

Question 37 - CMA 691 1-9 - Budget Methodologies

The most direct way to prepare a cash budget for a manufacturing firm is to include

 A. Projected sales, credit terms, and net income.B. Projected net income, depreciation, and goodwill impairment.

Part 1 : 07/28/10 08:28:23

(c) HOCK international, page 24

Page 25: Budget Methodologies

8/20/2019 Budget Methodologies

http://slidepdf.com/reader/full/budget-methodologies 25/61

C. Projected purchases, percentages of purchases paid, and net income.D. Projected sales and purchases, percentages of collections, and terms of payments.

 A. Net income is not included in the cash budget because it includes noncash items like depreciation and theamortization of bond premium.

B. Projected net income is not included in the cash budget because it includes noncash items like depreciation andthe impairment of goodwill. Depreciation and goodwill impairment are not included because they are noncash items.

C. Net income is not included in the cash budget because it includes noncash items like depreciation and theamortization of bond premium.

D. All of these items listed are related to cash and would therefore be included in the preparation of a cashbudget.

Question 38 - CMA 1292 H2 - Budget Methodologies

Butteco has the following cost components for 100,000 units of product for the year.

Raw materials $200,000

Direct labor 100,000

Manufacturing overhead 200,000

Selling/administrative expense 150,000

 All costs are variable except for $100,000 of manufacturing overhead and $100,000 of selling and administrativeexpenses. The total costs to produce and sell 110,000 units are

 A. $715,000.B. $695,000.

C. $495,000.D. $650,000.

 A. This answer assumes no fixed costs.

B. In order to solve this problem we need to determine a total fixed cost and the variable cost per un it. Thetotal f ixed cos ts are $200,000 ($100,000 each of manufacturing and selling costs ). The total variable cos ts inthe 100,000 unit budget are $450,000 ($200,000 raw materials + 100,000 direct labor + $100,000manufacturing overhead + $50,000 selling/administrative expense). This gives a standard variable cost o f$4.50 per unit. Therefore, to produce 110,000 units the company w ill incur $495,000 in variable costs ($4.50 ×110,000 units) p lus $200,000 in fixed cos ts for a total of $695,000.

C. This is the variable cost of production only.

D. This is the cost at a production level of 100,000 units.

Question 39 - CMA 693 3-10 - Budget Methodologies

 A firm develops an annual cash budget in order to

 A. Support the preparation of its cash flow statement for the annual report.B. Ascertain which capital expenditure projects are feasible and which capital expenditure projects should bedeferred.

C. Determine the opportunity costs of alternative sales and production strategies.D. Avoid the opportunity costs of non-invested excess cash and minimize the cost of interim financing.

Part 1 : 07/28/10 08:28:23

(c) HOCK international, page 25

Page 26: Budget Methodologies

8/20/2019 Budget Methodologies

http://slidepdf.com/reader/full/budget-methodologies 26/61

 A. The cash flow statement in the annual report is based on actual results, not on budgeted figures.

B. The cash budget does not ascertain which capital expenditure projects are feasible or should be deferred. It onlyshows the cash available for projects and other activities.

C. The cash budget does not determine the opportunity costs of alternative sales and production strategies.

D. The cash budget (or cash management and work ing capital budget) is the last budget created. The cashbudget tracks the inflows and outflows of cash on a month-by-month (possibly even week-by-week orday-by-day) basis. If this budget is accurate it will allow t he company to plan for any cash shortfalls thatmay occur during the year and also enable it to plan for any excess cash accumu lating during the year. Oneadvantage of predicting cash shor tfalls is that it w ill be easier for the company to obtain a loan if it is awareof its need before the shortfall arrives and if it is able to present cash inflow and outf low pro jections to thebank.

Question 40 - CMA 1291 3-13 - Budget Methodologies

 A flexible budget is appropriate for 

 A. Control of direct materials and direct labor but not selling and administrative expenses.B. Control of direct labor and direct materials but not fixed factory overhead.C. Any level of activity.D. Control of fixed factory overhead but not direct materials and direct labor.

 A. A flexible budget is necessary to control direct materials and direct labor as well as variable selling andadministrative expenses if the actual activity level differs from the static budget activity level. To control fixed costs,the use of static budget is appropriate, not necessarily the use of flexible budget.

B. Fixed cost s are the same for any level of activit y within the relevant range. Thus, a flexible budget is notnecessary to control fixed factory overhead. In fact, total fixed cos ts are the same in a flexible budget as

they are in the static budget. However, a flexible budget is necessary to contro l direct materials and directlabor if the actual activity level differs from the static budget activity level.

C. While a flexible budget can be prepared for any level of activity, this is not the best answer to the question amongthe answer choices given.

D. Fixed costs are the same for any level of activity within the relevant range. Thus, a flexible budget is not necessaryto control f ixed factory overhead. A flexible budget is necessary to control direct materials and direct labor if theactual activity level differs from static budget activity level.

Question 41 - CMA 692 3-8 - Budget Methodologies

The budget that is usually the most difficult to forecast is the

 A. Production budget.B. Expense budget.C. Manufacturing overhead budget.D. Sales budget.

 A. The production budget is based on the sales budget, and once the sales budget has been prepared and endinginventory objectives determined, the production budget is relatively easy to prepare.

B. There is no such thing as an "expense budget." General and administrative expenses are budgeted as part of the

operating budget, and they are based upon management's plans for the coming year. Selling expenses are also apart of the operating budget, and they are based on the sales budget.

Part 1 : 07/28/10 08:28:23

(c) HOCK international, page 26

Page 27: Budget Methodologies

8/20/2019 Budget Methodologies

http://slidepdf.com/reader/full/budget-methodologies 27/61

C. The preparation of the manufacturing overhead budget is fairly straight-forward after the production budget hasbeen prepared, because it is based on the production budget.

D. The sales budget is usually the first budget to be prepared, and it influences all other budgets in themaster budget. The volume of sales and level of sales revenue are difficult to forecast, as many externalfactors can influence this budget. The sales budget is based on a number of assumptions about thechanging environment such as competitors' activities, consumer tastes and demand, prices, the generaleconomy, government regulations, and many other factors that are outside the control of the company.Therefore, the sales budget is very difficu lt to develop.

Question 42 - CIA 1192 IV-19 - Budget Methodologies

There are many different budget techniques or processes that business organizations can employ. One of thesetechniques or processes is zero-base budgeting, which is

 A. Budgeting from the ground up as though the budget process were being initiated for the first time.B. Developing budgeted costs from clear-cut measured relationships between inputs and outputs.

C. Using the prior year's budget as a base year and adjusting it based on the experiences of the prior year and theexpectations for the coming year.D. Budgeting for cash inflows and outflows to time investments and borrowings in a way to maintain a bank accountwith a minimum balance.

 A. Zero-based budgeting is the budgeting method in which the cu rrent year budget is prepared w ithout anyreference to, or use of, the prior period's budget. Within zero-based budgeting all of the activit ies that adepartment undertakes are identified and then prio ritized. This is the manner in which costs are justifiedand supported. Also, because the manager needs to examine every single expenditu re and activity wi thinthe department, he is more likely to develop an alternative and, hopefully cheaper, method of accomplishingthe same thing.

B. This answer is incorrect. See the correct answer for a complete explanation.

C. This refers to incremental budgeting.

D. This answer is incorrect. See the correct answer for a complete explanation.

Question 43 - CIA 1190 IV-15 - Budget Methodologies

 A company has budgeted sales of 24,000 finished units for the forthcoming 6-month period. It takes 4 pounds ofdirect materials to make one finished unit. Given the following:

 Finished Units

Direct Materials(pounds)

Beginning inventory 14,000 44,000

Target ending inventory 12,000 48,000

How many pounds of direct materials should be budgeted for purchase during the 6-month period?

 A. 48,000B. 88,000C. 96,000D. 92,000

 A. This is the amount of ending inventory of materials. See the correct answer for a complete explanation.

B. This is the amount of materials requirements for the period's production. See the correct answer for a complete

Part 1 : 07/28/10 08:28:23

(c) HOCK international, page 27

Page 28: Budget Methodologies

8/20/2019 Budget Methodologies

http://slidepdf.com/reader/full/budget-methodologies 28/61

explanation.

C. This answer is incorrect. See the correct answer for a complete explanation.

D. First, we need to determine the produc tion requirements f or the given period. It is calculated as follow s:Units Sold + Ending Inventory - Beginning Inventory ; or 24,000 + 12,000 - 14,000 = 22,000 units to p roduce.We know that 4 lb. is necessary to make one unit of fi nished product . Thus, materials to be purchasedequals the Quantity needed for production + Ending Inventory of materials - Beginning Inventory ofmaterials; or 22,000 * 4 + 48,000 - 44,000 = 92,000.

Question 44 - CMA 1290 3-17 - Budget Methodologies

The operating budget process usually begins with the

 A. Financial budget.B. Sales budget.C. Income statement.

D. Balance sheet.

 A. The financial budget is neither an operating budget nor a part of the operating budget.

B. The sales budget is a part of operating budget and is usually the firs t budget to be prepared in thebudgeting process. It is therefore the foundation of the operating budget.

C. The pro forma income statement is a part of the financial budget, not the operating budget.

D. The pro forma balance sheet is a part of the financial budget, not the operating budget.

Question 45 - CMA 1293 3-11 - Budget Methodologies

Superflite expects April sales of its deluxe model airplane, the C-14, to be 402,000 units at $11 each. Each C-14requires three purchased components shown below.

 Purchase

CostNumber Needed

for each C-14 Unit

 A-9 $0.50 1

B-6 0.25 2

D-28 1.00 3

Factory direct labor and variable overhead per unit of C-14 totals $3.00. Fixed factory overhead is $1.00 per unit at aproduction level of 500,000 units. Superflite plans the following beginning and ending inventories for the month of

 April and uses standard absorption costing for valuing inventory.

Part No.Units at April 1

Units at April 30

C-14 12,000 10,000

 A-9 21,000 9,000

B-6 32,000 10,000

D-28 14,000 6,000

 Assume Superflite plans to manufacture 400,000 units in April. Superflite's April budget for the purchase of A-9

should be

Part 1 : 07/28/10 08:28:23

(c) HOCK international, page 28

Page 29: Budget Methodologies

8/20/2019 Budget Methodologies

http://slidepdf.com/reader/full/budget-methodologies 29/61

 A. 379,000 units.B. 388,000 units.C. 402,000 units.D. 412,000 units.

 A. This number does not take ending inventory level into consideration. See the correct answer for a completeexplanation.

B.

To solve these question we should use the formula for the physical flow of inventory:

Beginning Inventory + Purchases  Units Used in Production = Ending Inventory

Since 1 unit of A-9 is required to produce one unit of fin ished goods, the number of unit s of A-9 needed forproduction will be the same as the number of units to be produced: 400,000. The number of units inbeginning and ending inventory are given as 21,000 and 9,000, respectively. Therefore, the formula will be:

21,000 + Purchases  400,000 = 9,000

Solving for Purchases, we get Purchases = 388,000 units .

C. This is the number of unit sales of finished product. See the correct answer for a complete explanation.

D. This result is based on an incorrect inventory formula, using the ending inventory as the beginning inventory andthe beginning inventory as ending inventory. See the correct answer for a complete explanation.

Question 46 - CIA 1190 IV-16 - Budget Methodologies

 A company is preparing its cash budget for the coming month. All sales are made on account. Given the following:

 BeginningBalances

Budgeted Amounts

Cash $50,000

 Accounts receivable 180,000

Sales $800,000

Cash disbursements 780,000

Depreciation 25,000

Ending accounts receivable balance 210,000

What is the expected cash balance of the company at the end of the coming month?

 A. $70,000B. $45,000C. $15,000D. $40,000

 A. This answer is incorrect. See the correct answer for a complete explanation.

B. This answer is incorrect. See the correct answer for a complete explanation.

C. In this answer the amount of depreciation is deducted. However, depreciation is a non-cash expense and is notincluded in the calculation of expected cash balance. See the correct answer for a complete explanation.

D. The cash balance at the end of the period is equal to the: Beginning Cash balance + Sales - Ending AR +Beginning AR - Cash Disbursements or $50,000 + $800,000 - $210,000 + $180,000 - $780,000 = $40,000.Depreciation is non-cash expense and is not included in calculation.

Part 1 : 07/28/10 08:28:23

(c) HOCK international, page 29

Page 30: Budget Methodologies

8/20/2019 Budget Methodologies

http://slidepdf.com/reader/full/budget-methodologies 30/61

Question 47 - CMA 692 H2 - Budget Methodologies

Pardise Company budgets on an annual basis for its fiscal year. The following beginning and ending inventory levels

(in units) are planned for the fiscal year of July 1 through June 30:

  July 1 June 30

Raw material* 40,000 50,000

Work-in-process 10,000 20,000

Finished goods 80,000 50,000

*Two units of raw materials are needed to produce each unit of finished product.

If Pardise Company plans to sell 480,000 units during the fiscal year, the number of units it will have to manufactureduring the year is

 A. 440,000 units.

B. 450,000 units.C. 510,000 units.D. 480,000 units.

 A. The change in work-in-process does not need to be included in this calculation.

B. If Paradise will sell 480,000 units and they want to have 50,000 units in ending f inished goods inventory,Paradise wi ll need to have 530,000 units during the period. Since they already have 80,000 of these units inbeginning inventor y, they will need to manufacture 450,000 units i n order to be able to meet the sales andending inventory plans.

C. This answer treats the beginning and ending finished goods inventory backwards.

D. This is the number of units to be sold.

Question 48 - CMA 1293 3-10 - Budget Methodologies

Superflite expects April sales of its deluxe model airplane, the C-14, to be 402,000 units at $11 each. Each C-14requires three purchased components shown below.

 Purchase

CostNumber Needed

for each C-14 Unit

 A-9 $0.50 1

B-6 0.25 2

D-28 1.00 3

Factory direct labor and variable overhead per unit of C-14 totals $3.00. Fixed factory overhead is $1.00 per unit at aproduction level of 500,000 units. Superflite plans the following beginning and ending inventories for the month of

 April and uses standard absorption costing for valuing inventory.

Part No.Units at April 1

Units at April 30

C-14 12,000 10,000

 A-9 21,000 9,000

B-6 32,000 10,000D-28 14,000 6,000

Part 1 : 07/28/10 08:28:23

(c) HOCK international, page 30

Page 31: Budget Methodologies

8/20/2019 Budget Methodologies

http://slidepdf.com/reader/full/budget-methodologies 31/61

The C-14 production budget for April should be based on the manufacture of 

 A. 402,000 units.B. 424,000 units.C. 400,000 units.D. 390,000 units.

 A. This is the number of unit sales. It does not take the change in inventory level into consideration. See the correctanswer for a complete explanation.

B. This answer is incorrect. See the correct answer for a complete explanation.

C.

To solve these question we should use the formula of phys ical flow of inventory:

Beginning Inventory + Units Produced  Units Sold = Ending Inventory.

Plugging numbers for C-14 into the formula, we get the following:

12,000 + Units Produced  402,000 = 10,000

Solving for Units Produced, we get Units Produced = 400,000 units.

D. This answer ignores the requirements to have some ending inventory on hand. See the correct answer for acomplete explanation.

Question 49 - CMA 1292 H1 - Budget Methodologies

When preparing a performance report for a cost center using flexible budgeting techniques, the planned cost

column should be based on the

 A. Budgeted amount in the original budget prepared before the beginning of the year.B. Budget adjusted to the planned level of activity for the period being reported.C. Budget adjusted to the actual level of activity for the period being reported.D. Actual amount for the same period in the preceding year.

 A. This answer is incorrect. See the correct answer for a complete explanation.

B. This answer is incorrect. See the correct answer for a complete explanation.

C. The flexible budget is the budget developed for the actual level of output . When preparing a performancereport, the actual results need to be compared to what the expected results were for the actual level of

production. The actual results need to be compared to the flexible budget.

D. This answer is incorrect. See the correct answer for a complete explanation.

Question 50 - CIA 589 IV-12 - Budget Methodologies

 A company has $10,000 in cash and $150,000 in merchandise inventory on March 31. The desired cash andmerchandise inventory balances on June 30 are $20,000 and $250,000, respectively. Sales for the quarter areexpected to be $300,000, all in cash. Gross margin is 40% of sales. Cash operating expenses are expected to be$50,000. All merchandise inventory purchases are paid for in cash at the time of purchase. What amount of

financing will the company need during the quarter?

 A. $40,000

Part 1 : 07/28/10 08:28:23

(c) HOCK international, page 31

Page 32: Budget Methodologies

8/20/2019 Budget Methodologies

http://slidepdf.com/reader/full/budget-methodologies 32/61

B. $30,000C. $20,000D. $50,000

 A. Firs t, we need to determine the amount of purchases dur ing the quar ter . Beginning merchand iseinvento ry level is $150,000 and ending merchandise inventory is $250,000. The cost of sales expected to bemade during the quarter is $180,000 ($300,000 in sales × 60%). Thus, the purchases are equal to $280,000($180,000 cost of sales + $250,000 ending inventory  $150,000 beginning inventory). Now w e can determinethe amount of f inancing the company w ill need during the quarter. It is equal to Purchases + Expenses +Ending cash balance  Beginning cash balance  Proceeds f rom sales. We get : $280,000 + $50,000 + $20,000 $10,000  $300,000 = $40,000.

B. This answer does not include the beginning or ending cash balance. See the correct answer for a completeexplanation.

C. This answer does not include the ending cash balance. See the correct answer for a complete explanation.

D. This answer does not include the beginning cash balance. See the correct answer for a complete explanation.

Question 51 - CMA 692 3-26 - Budget Methodologies

Berol Company plans to sell 200,000 units of finished product in July and anticipates a growth rate in sales of 5% permonth. The desired monthly ending inventory in units of finished product is 80% of the next month's estimated sales.There are 150,000 finished units in inventory on June 30. Each unit of finished product requires 4 pounds of directmaterials at a cost of $1.20 per pound. There are 800,000 pounds of direct materials in inventory on June 30.

 Assume Berol Company plans to produce 600,000 units of finished product in the 3-month period ending September30, and to have direct materials inventory on hand at the end of the 3-month period equal to 25% of the use in thatperiod. The estimated cost of direct materials purchases for the 3-month period ending September 30 is

 A. $2,640,000.B. $2,400,000.C. $2,200,000.D. $2,880,000.

 A.

First, we need to identif y the quantit y of material that needs to be purchased, then calculate its cost.

The beginning inventory level on Ju ly 1 is 800,000 pounds. Since each unit of fi nished goods requires 4pounds of direct materials, the amount of d irect materials to be used in production is 600,000 × 4, or2,400,000 pounds.The amount of di rect materials inventory on hand at the end of the 3-month period is to beequal to 25% of the use in that period. Therefore, ending inventory needs to be 600,000 pounds o f material

(2,400,000 lb. × 25%).

Now we can determine the quantity to be purchased:

Beginning Inventory + Quantity Purchased  Quantity Used in Production = Ending Inventory

800,000 + Quantity Purchased  2,400,000 = 600,000

Solving for Quantity Purchased,

Quantity Purchased = 2,200,000 pounds.

Given a cost of $1.20 per pound of material, we can determine the cost of materials to be purchased is

$2,640,000 ($1.20 × 2,200,000).

B. This is the number of pounds that will be used. See the correct answer for a complete explanation.

Part 1 : 07/28/10 08:28:23

(c) HOCK international, page 32

Page 33: Budget Methodologies

8/20/2019 Budget Methodologies

http://slidepdf.com/reader/full/budget-methodologies 33/61

C. This is the number of pounds that needs to be purchased. See the correct answer for a complete explanation.

D. This answer is incorrect. See the correct answer for a complete explanation.

Question 52 - CMA 1287 4-29 - Budget Methodologies

The Jung Corporation's budget calls for the following production:

Qtr 1 : 45,000 unitsQtr 2 : 38,000 unitsQtr 3 : 34,000 unitsQtr 4 : 48,000 units

Each unit of product requires three pounds of direct material. The company's policy is to begin each quarter with aninventory of direct materials equal to 30% of that quarter's direct material requirements. Budgeted direct materialspurchases for the third quarter would be

 A. 114,600 pounds.B. 43,200 pounds.C. 30,600 pounds.D. 38,200 pounds.

 A.

This is a basic question of unit s needed in a period, but it is about the number of un its of the raw materialsthat are needed. There are 3 units of raw materials in a finished unit. The amount needed in the third quarteritself is 102,000 units of raw materials (3 * 34,000 finished unit s). In addition, the ending inventor y is 30% ofthe next quarter's needs. This is 43,200 units of raw materials (48,000 finished unit s * 3 * .3). The beginninginvento ry was 30% of the current quarter's needs or 30,600 (102,000 * .3).

Beginning Inventory + Purchases  Amount Used = Ending Inventory

30,600 + Purchases  102,000 = 43,200Purchases = 114,600

 A total of 114,600 uni ts of raw materials need to be purchased this period .

B. This is the ending inventory amount. See the correct answer for a complete explanation.

C. This is the amount of beginning inventory. See the correct answer for a complete explanation.

D. This answer is incorrect. See the correct answer for a complete explanation.

Question 53 - CMA 1295 H3 - Budget Methodologies

 All of the following are considered operating budgets except the

 A. Materials budget.B. Production budget.C. Capital budget.D. Sales budget.

 A. The materials budget is a part of the operating budget.

B. The production budget is a part of the operating budget.

Part 1 : 07/28/10 08:28:23

(c) HOCK international, page 33

Page 34: Budget Methodologies

8/20/2019 Budget Methodologies

http://slidepdf.com/reader/full/budget-methodologies 34/61

C. The capital budget is usually prepared separately from the other budgets. The capital budget concernscapital investments in p lant, equipment, real-estate, etc. Management plans these type of investments inadvance to allocate or obtain enough resources to perform them.

D. The sales budget is the first budget developed in the operating budget.

Question 54 - IMA 08-P2-11 - Budget Methodologies

Rock Industries has four divisions. In the quest to develop a more achievable budget for the coming year, the chiefexecutive officer has elected to develop the company's budget by using a decentralized bottom-up budget approach.Chip Jones is production manager in one of the divisions. Jones’ involvement in the budget process this year willprobably:

 A. Require development of a production budget that is forwarded to the Budget Department.B. Require development of a production budget based on the prior year’s manufacturing activity.C. Be negligible.D. Require development of a production budget after receiving the division’s projected sales forecast.

 A. This is incorrect. Jones will develop a production budget that is forwarded to the Budget Department; however,first he will have to have an idea of how much to produce. This will come from the sales forecast.

B. While prior year activity is one place to start, it does not include the sales forecast for the upcoming year oradjustments for changes in costs or processes.

C. In a bottom up budget, the department managers will have a significant role in budget development.

D. Jones wi ll develop a budget for his production department after determining how much is expected to besold. The production budget will take into account what is to be sold as well as cushions for extra sales andprovisions for ending finished goods.

Question 55 - CMA 692 H7 - Budget Methodologies

Which one of the following may be considered an independent item in the preparation of the master budget?

 A. Capital investment budget.B. Ending inventory budget.C. Budgeted statement of financial position.D. Budgeted income statement.

 A. The capi tal budget is the budget in which al l capi tal (property, plant and equipment ) expend itures are

planned. This budget is not directly connected to the current period budgets and it is often prepared yearsin advance so that the company can plan to obtain the necessary financing or accumulate the necessarycash to carry out its capital expansion plans. The capital budget is often considered to be independent fromthe master budget.

B. The desired level of ending inventory not independent, as it is used in the development of the production budget.

C. The budgeted statement of financial position (balance sheet) is based on a number of elements of the masterbudget including the budgeted income statement, so it is not an independent item in the preparation of the masterbudget.

D. The budgeted income statement is a critical element of the master budget. It is one of the last budgets createdfrom the operating budgets. The budgeted income statement is based on the sales budgets, expense budgets and

other elements of the master budget, so it is not an independent item in the preparation of the master budget.

Part 1 : 07/28/10 08:28:23

(c) HOCK international, page 34

Page 35: Budget Methodologies

8/20/2019 Budget Methodologies

http://slidepdf.com/reader/full/budget-methodologies 35/61

Page 36: Budget Methodologies

8/20/2019 Budget Methodologies

http://slidepdf.com/reader/full/budget-methodologies 36/61

 A.

The budgeting preparation process usually starts with the sales budget and continues through thepreparation of the income statement wh ich is the last budget of the operating budget and ends withpreparation of the financial budget, which includes the budgeted balance sheet and statement of cashflows, the cash budget and the capital budget.

The cash budget is a part of the financial budget and is one of the last budgets to be prepared. This isbecause the cash budget is impacted by all of the other individual budgets.

B. The manufacturing overhead budget is an operating budget and it is completed prior to the cash budget.

C. The cost of goods sold budget is a part of the operating budget and it is completed prior to the cash budget.

D. The selling expense budget is an operating budget and it is completed prior to the cash budget.

Question 59 - CMA 1294 H2 - Budget Methodologies

Of the following items, the one item that would not be considered in evaluating the adequacy of the budgeted annualoperating income for a company is

 A. Internal rate of return.B. Price-earnings ratio.C. Earnings per share.D. Industry average for earnings on sales.

 A. The internal rate o f retu rn is used to evaluate investment decisions and involves the time value of money.IRR represents the discoun t rate at which the net present value of an investment is equal to zero. IRR is notused to evaluate the adequacy of budgeted operating income.

B. The price-earnings ratio is a financial performance measure and can be used to measure the adequacy of thebudgeted annual operating income.

C. Earning per share represents a financial performance measure and can be used to measure the adequacy of thebudgeted annual operating income.

D. The industry average for earnings on sales is a financial performance measure and can be used to measure theadequacy of the budgeted annual operating income.

Question 60 - CMA 1293 H3 - Budget Methodologies

The Raymar Company is preparing its cash budget for the months of April and May. The firm has established a$200,000 line of credit with its bank at a 12% annual rate of interest on which borrowings for cash deficits must bemade in $10,000 increments. There is no outstanding balance on the line of credit loan on April 1. Principalrepayments are to be made in any month in which there is a surplus of cash. Interest is to be paid monthly. If thereare no outstanding balances on the loans, Raymar will invest any cash in excess of its desired end-of-month cashbalance in U.S. Treasury bills. Raymar intends to maintain a minimum balance of $100,000 at the end of eachmonth by either borrowing for deficits below the minimum balance or investing any excess cash. Monthly collectionand disbursement patterns are expected to be:

Collections. 50% of the current month's sales budget and 50% of the previous month's sales budget.

 Accounts Payable Disbursements. 75% of the current month's accounts payable budget and 25% of theprevious month's accounts payable budget.

 All other disbursements occur in the month in which they are budgeted.

Budget Information

Part 1 : 07/28/10 08:28:23

(c) HOCK international, page 36

Page 37: Budget Methodologies

8/20/2019 Budget Methodologies

http://slidepdf.com/reader/full/budget-methodologies 37/61

  March April May

Sales $40,000 $50,000 $100,000

 Accounts payable 30,000 40,000 40,000

Payroll 60,000 70,000 50,000

Other disbursements 25,000 30,000 10,000

In April, Raymar's budget will result in

 A. A need to borrow $100,000 on its line of credit for the cash deficit.B. $45,000 in excess cash.C. A need to borrow $90,000 on its line of credit for the cash deficit.D. A need to borrow $50,000 on its line of credit for the cash deficit.

 A.

First, we need to determine the cash co llections f or April: 50% of April sales and 50% of March sales (or$25,000 + $20,000 = $45,000) wil l be col lected in April . Then, we need to determine the amount paid foraccounts payable in Apr il: 75% of April AP and 25% of March AP (or $30,000 + $7,500 = $37,500) will be paid

in April. Other disbursements are paid in the month they occur, and for April they are: $70,000 for payro llplus $30,000 of other disbursements, totaling $100,000. Subtracting the amount of cash outflow s from cashinflow s we wi ll get a $92,500 cash defici t. Since borrowings for cash defic its must be made in $10,000increments the company needs to bo rrow $100,000 to cover the $92,500 cash defici t.

B. This is the amount of cash collections only. See the correct answer for a complete explanation.

C. Borrowings for cash deficits must be made in $10,000 increments the company needs to borrow $100,000 tocover $92,500 of cash deficit.

D. This answer is incorrect. See the correct answer for a complete explanation.

Question 61 - IMA 08-P2-40 - Budget Methodologies

Streeter Company produces plastic microwave turntables. Sales for the next year are expected to be 65,000 units inthe first quarter, 72,000 units in the second quarter, 84,000 units in the third quarter, and 66,000 units in the fourthquarter. Streeter maintains a finished goods inventory at the end of each quarter equal to one half of the unitsexpected to be sold in the next quarter. How many units should Streeter produce in the second quarter?

 A. 84,000 unitsB. 72,000 unitsC. 75,000 unitsD. 78,000 units

 A. This is the total sales budgeted for the third quarter. See correct answer for full calculation.

B. This is the sales for the second quarter. Beginning and ending inventory need to be taken into consideration. Seecorrect answer for full calculation.

C. This is the number of units Streeter should produce in the third quarter.

D. The second quarter's beginning inventory will be 50% of the 72,000 units to be so ld during the secondquarter, or 36,000 units.

The second quarter's end ing inventory w ill be 50% of the 84,000 units to be sold during the third quarter, or42,000 units.

Units to be sold during the second quarter are given as 72,000.

Part 1 : 07/28/10 08:28:23

(c) HOCK international, page 37

Page 38: Budget Methodologies

8/20/2019 Budget Methodologies

http://slidepdf.com/reader/full/budget-methodologies 38/61

The inventory equation is:

Beginning Inventory + Goods Manufactured – Goods Sold = Ending Inventory

Let X represent Goods Manufactured:

36,000 + X – 72,000 = 42,000

X = 78,000

Question 62 - IMA 08-P2-05 - Budget Methodologies

The following sequence of steps are employed by a company to develop its annual profit plan.

Planning guidelines are disseminated downward by top management after receiving input from all levels ofmanagement.

 A sales budget is prepared by individual sales units reflecting the sales targets of the various segments. This

provides the basis for departmental production budgets and other related components by the variousoperating units. Communication is primarily lateral with some upward communication possible.

 A profit plan is submitted to top management for coordination and review. Top management'srecommendations and revisions are acted upon by middle management. A revised profit plan is resubmittedfor further review to top management.

Top management grants final approval and distributes the formal plan downward to the various operatingunits.

This outline of steps best describes which one of the following approaches to budget development?

 A. Total justification of all activities by operating units.B. Bottom-up approach.C. Imposed budgeting by top management.

D. Top-down approach.

 A. This would describe zero based budgeting and is not described by the activities listed above. With zero basedbudgeting, every line item is analyzed and justified in a time consuming process. The guidelines developed frommanagerial input do not indicate such a thorough analysis.

B. While senior management provides guidelines, the bulk of the budget is developed from input fromindiv idual departments. After submission to top management for review, top management'srecommendations and revisions are made at the individual department level, and the indiv idual departmentmanagers prepare and resubmit revised budget data to top management. This continues until the desiredresult is achieved for the consolidated budget.

C. Management did not dictate the budget, just provided guidelines to give each department an idea of where the

company should be headed.

D. Management did not provide the budgeted numbers, just made recommendations for revisions to the budgetssubmitted by the various departments.

Question 63 - CIA 585 III-20 - Budget Methodologies

The major feature of zero-based budgeting (ZBB) is that it

 A. Focuses on planned capital outlays for property, plant, and equipment.

B. Assumes all activities are legitimate and worthy of receiving budget increases to cover any increased costs.C. Takes the previous year's budgets and adjusts them for inflation.D. Questions each activity and determines whether it should be maintained as it is, reduced, or eliminated.

Part 1 : 07/28/10 08:28:23

(c) HOCK international, page 38

Page 39: Budget Methodologies

8/20/2019 Budget Methodologies

http://slidepdf.com/reader/full/budget-methodologies 39/61

 A. This is usually a major feature of a capital budget.

B. Incremental or traditional budget usually assumes that all activities are legitimate and worthy of receiving budgetincreases to cover any increased costs. In ZBB all activities need to be justified each year.

C. Incremental budgeting usually takes the previous period budget and make adjustments to develop a budget for a

following period.

D. Zero-based budgeting is the budgeting method in which the current year budget is prepared for variouslevels of service that may be provided without any reference to, or use of, the prior period's budget. Thoughthis method is more time consuming and dif ficult fo r all of the people involved, there are a number ofadvantages to the company as a result of using th is method. Because the budget is built up from zero, eachmanager must just ify all of the expenses in his o r her department. Each component is evaluated from acost -benefit perspective and prioriti es are made. Zero-based budgeting enables the company to identifyexpenses that are not value adding or that shou ld be reduced due to some development in produc tionmethods or s omething similar.

Question 64 - CMA 1296 H11 - Budget Methodologies

Daffy Tunes manufactures a toy rabbit with moving parts and a built-in voice box. Projected sales in units for the next5 months are as follows:

Month Sales in Units

January 30,000

February 36,000

March 33,000

 April 40,000

May 29,000

Each rabbit requires basic materials that Daffy purchases from a single supplier at $3.50 per rabbit. Voice boxes arepurchased from another supplier at $1.00 each. Assembly labor cost is $2.00 per rabbit, and variable overhead costis $.50 per rabbit. Fixed manufacturing overhead applicable to rabbit production is $12,000 per month. Daffy's policyis to manufacture 1.5 times the coming month's projected sales every other month, starting with January (i.e.,odd-numbered months) for February sales, and to manufacture 0.5 times the coming month's projected sales inalternate months (i.e., even-numbered months). This allows Daffy to allocate limited manufacturing resources toother products as needed during the even-numbered months.

The dollar production budget for toy rabbits for February is

 A. $113,500B. $390,000C. $327,000D. $127,500

 A. This is production budget for April. See the correct answer for a complete explanation.

B. This is production budget for January. See the correct answer for a complete explanation.

C. These are the costs based on production of 150% of January sales. See the correct answer for a completeexplanation.

D. In February the production w ill be equal to 1/2 of March sales. March sales are expected to be 33,000, soFebruary w ill see production of 16,500 uni ts. The variable cost per uni t is $7 ($3.50 + $1 + $2 + $.50), so to talvariable cos ts w ill be $115,500 ($7 × 16,500). We need to add to this the $12,000 of fixed costs giving us atotal product ion cost of $127,500.

Part 1 : 07/28/10 08:28:23

(c) HOCK international, page 39

Page 40: Budget Methodologies

8/20/2019 Budget Methodologies

http://slidepdf.com/reader/full/budget-methodologies 40/61

Question 65 - CMA 1294 H5 - Budget Methodologies

When sales volume is seasonal in nature, certain items in the budget must be coordinated. The three mostsignificant items to coordinate in budgeting seasonal sales volume are

 A. Production volume, finished goods inventory, and sales volume.B. Direct labor hours, work-in-process inventory, and sales volume.C. Raw material inventory, work-in-process inventory, and production volume.D. Raw material inventory, direct labor hours, and manufacturing overhead costs.

 A. Budgets usually st art with the development of the sales budget, and it is the most d if ficu lt budget toprepare. Production volume and finished goods inventory budgets are the next budgets to be prepared, andtheir development is based on the assumptions made in preparing the sales budget. When the business isseasonal in nature it is even more difficu lt to predict sales volume and subsequent budgets. Thus, for anytype of business including seasonal it is critical to coordinate production volume, finished goods inventory,and sales volume first o f all.

B. Direct labor hours, work-in-process inventory are elements within the production budget. See the correct answerfor a complete explanation.

C. The sales budget is usually the first thing to determine in the budgeting process of any type of business includinga seasonal business. See the correct answer for a complete explanation.

D. The sales budget is usually the first thing to determine in the budgeting process of any type of business includinga seasonal business. See the correct answer for a complete explanation.

Question 66 - CMA 1283 4-23 - Budget Methodologies

Kelly Company is a retail sporting goods store that uses accrual accounting for its records. Facts regarding Kelly's

operations are as follows:Sales are budgeted at $220,000 for December year 1 and $200,000 for January year 2.

Collections are expected to be 60% in the month of sale and 38% in the month following the sale.

Gross margin is 25% of sales.

 A total of 80% of the merchandise held for resale is purchased in the month prior to the month of sale and20% is purchased in the month of sale. Payment for merchandise is made in the month following thepurchase.

Other expected monthly expenses to be paid in cash are $22,600.

 Annual depreciation is $216,000.

Below is Kelly Company's statement of financial position at November 30, year 1.

 Assets  

Cash $22,000

 Accounts receivable(net of $4,000 allowance for uncollectible accounts)

76,000

Inventory 132,000

Property, plant, and equipment (net of $680,000 accumulated deprecation) 870,000

Total assets $1,100,000

Liabilities and Stockholders' Equity  

 Accounts payable $162,000

Common stock 800,000

Retained earnings 138,000

Total liabilities and stockholders' equity $1,100,000

Part 1 : 07/28/10 08:28:23

(c) HOCK international, page 40

Page 41: Budget Methodologies

8/20/2019 Budget Methodologies

http://slidepdf.com/reader/full/budget-methodologies 41/61

The budgeted income (loss) before income taxes for December year 1 is

 A. $28,000.B. $32,400.C. $10,000.D. Some amount other than those given.

 A. This amount does not consider depreciation expenses. See the correct answer for a complete explanation.

B. This amount does not consider bad debt and depreciation expenses. See the correct answer for a completeexplanation.

C. The gross margin is $55,000 ($220,000 × 25%). Subtracting other expenses, bad debt expenses ($220,000× 2%) and month ly depreciation ($216,000 ÷ 12) from gross margin results in a budgeted net income of$10,000 ($55,000  $22,600  $4,400  $18,000).

D. The correct amount is given as one of the choices.

Question 67 - CMA 1290 3-19 - Budget Methodologies

The use of the master budget throughout the year as a constant comparison with actual results signifies that themaster budget is also a

 A. Zero-base budget.B. Capital budget.C. Flexible budget.D. Static budget.

 A. In a zero-base budget the current year's budget is prepared without any reference to, or use of, the prior period'sbudget. Not all master budgets are zero-base budgets, and the fact that the master budget is used to make

comparisons with actual results does not mean that it must be a zero-based budget.

B.

The capital budget is the budget in which all capital (property, plant and equipment) expenditures are planned. Thecapital budget is often prepared for several years in advance. The amounts in it that affect the current year's budgetwill be included in the current year's master budget; but the master budget is not also the capital budget.

C. A flexible budget is prepared for the actual level of activity, so it is not constant throughout the year.

D. This is a descrip tion of a static budget because the static budget is prepared for only one level of activity(a certain level of sales).

Question 68 - CMA 1289 4-25 - Budget Methodologies

Birch Corporation has the following historical pattern on its credit sales.

70% collected in month of sale15% collected in the first month after sale10% collected in the second month after sale4% collected in the third month after sale1% uncollectible

The sales on open account have been budgeted for the first 6 months of the year are as follows:

Part 1 : 07/28/10 08:28:23

(c) HOCK international, page 41

Page 42: Budget Methodologies

8/20/2019 Budget Methodologies

http://slidepdf.com/reader/full/budget-methodologies 42/61

MonthSales On

Open Account  

January $70,000

February 90,000

March 100,000

 April 120,000

May 100,000

June 90,000

The estimated total cash collections during the second calendar quarter from sales made on open account duringthe second calendar quarter would be

 A. $262,000B. $288,800C. $306,900D. $310,000

 A.

 Apr il , May and June are the months of the second quarter. The projected cash co llect ion in the secondquarter during the second quarter are:

70% of these three months' estimated credit sales,15% of credit sales made during April and May10% of c redit sales made dur ing May

Mathematically, th is looks as follows: 70% × ($120,000 + $100,000 +$90,000) + 15% × ($120,000 + $100,000) +10% × $120,000. Doing the math, we get $217,000 + $33,000 + $12,000 = $262,000.

This problem can also be solved by calculating each month's cash collection and then adding the resultstogether.

B. This answer is incorrect. See the correct answer for a complete explanation.

C. This answer is incorrect. See the correct answer for a complete explanation.

D. This is the amount of projected total sales in the second quarter, not the cash collection.

Question 69 - IMA 08-P2-13 - Budget Methodologies

Budgeting problems where departmental managers are repeatedly achieving easy goals or failing to achieve

demanding goals can be best minimized by establishing:

 A. Better communication whereby managers discuss budget matters daily with their superiors.B. Participative budgeting where managers pursue objectives consistent with those set by top management.C. Preventive controls.D. A policy that allows managers to build slack into the budget.

 A. Budget discussions on a daily basis are a bit excessive and not an efficient way to conduct business. Generallythere is a budget "season" where the development of the budget is the primary focus, and daily discussions are notunusual. After budget season; however, the communication between managers and their superiors would likely notfocus solely on budget matters, although it would cover variances from the budget.

B. This is correct. By allowing managers to participate in the budgeting process with the objectives ofsenior management in mind, the managers can evaluate the department and f ind the most optimal, andrealistic, means to achieve the targets.

Part 1 : 07/28/10 08:28:23

(c) HOCK international, page 42

Page 43: Budget Methodologies

8/20/2019 Budget Methodologies

http://slidepdf.com/reader/full/budget-methodologies 43/61

C. Enacting policies that "control" the budget process can result in resentment and ineffective efforts fromdepartmental management. This is not the best way to generate a realistically challenging budget.

D. Building slack into a budget is not going to be in the organization’s best interest in the long run.

Question 70 - CMA 1294 3-19 - Budget Methodologies

Superior Industries' sales budget shows quarterly sales for the next year as follows:

Quarter Units

1 10,000

2 8,000

3 12,000

4 14,000

Company policy is to have a finished goods inventory at the end of each quarter equal to 20% of the next quarter's

sales. Budgeted production for the second quarter of the next year would be

 A. 8,800 units.B. 8,400 units.C. 8,000 units.D. 7,200 units.

 A.

To solve these question we use the formula for the physical flow of goods:

Beginning Inventory + Units Produced  Units Sold = Ending Inventory

Beginning inventory for the second quarter is 20% of the second quarter sales, or 1,600 units . The endinginventor y for the second quarter is 20% if the third quarter sales, or 2,400 units. Plugging numbers into theformula we will get the following:

1,600 + Units Produced  8,000 = 2,400

Solving for Units Produced, we get Units Produced = 8,800.

B. This answer is incorrect. See the correct answer for a complete explanation.

C. This is the sales level of second quarter. See the correct answer for a complete explanation.

D. This answer is incorrect. See the correct answer for a complete explanation.

Question 71 - IMA 08-P2-10 - Budget Methodologies

Which one of the following statements concerning approaches for the budget development process is correct?

 A. With the information technology available, the role of budgets as an organizational communication device hasdeclined.B. To prevent ambiguity, once departmental budgeted goals have been developed, they should remain fixed even ifthe sales forecast upon which they are based proves to be wrong in the middle of the fiscal year.C. Since department managers have the most detailed knowledge about organizational operations, they should use

this information as the building blocks of the operating budget.D. The top-down approach to budgeting will not ensure adherence to strategic organizational goals.

Part 1 : 07/28/10 08:28:23

(c) HOCK international, page 43

Page 44: Budget Methodologies

8/20/2019 Budget Methodologies

http://slidepdf.com/reader/full/budget-methodologies 44/61

Page 45: Budget Methodologies

8/20/2019 Budget Methodologies

http://slidepdf.com/reader/full/budget-methodologies 45/61

 A. Is not used until the end of the budget period to evaluate performance.B. Overemphasizes a fixed time horizon such as one year.C. Makes across-the-board cuts when early budget iterations show that planned expenses are too high.D. Incorporates non-financial measures as well as financial measures into its output.

 A. In some organizations, this may occur. Actual performance for a period is measured against budgetedperformance for that period. While it is necessary to wait until the end of a period to measure the budgetary variancefor the whole period, the budget period can be broken into smaller timeframes. A 12 month budget can, and oftenwill, be divided into monthly amounts to allow for current month and year-to-date budget variance reportingthroughout the year, and operational adjustments can be made as necessary.

B. Most budgets do, in fact, focus on a one year, or 12 month time period. It is a strategic plan that looks farther intothe future.

C. The traditional budgeting process may lead to across-the-board cuts when early budget iterations show thatplanned expenses are too high. Budgeting should not necessarily require across the board cuts, even whenexpenses are higher than desired. Cost cutting should be based on what is best for the organization. Frequentlythose cuts will not be equally distributed.

D. This is not a criticism of the traditional budgeting process, because it is not a feature of any budgeting

process, traditional or non-traditional. A budget is quantitative and would not include non-financialmeasures in its output at all.

Question 74 - IMA 08-P2-28 - Budget Methodologies

 A budgeting approach that requires a manager to justify the entire budget for each budget period is known as:

 A. Incremental budgeting.B. Zero-base budgeting.C. Performance budgeting.

D. Program budgeting.

 A. Incremental budgeting simply takes historical information and adjusts it for anticipated increases or decreases inthe coming year. This type of budgeting does not involve justifying the entire budget for each budget period.

B. Zero-based budgeting is a budgeting method in which the budget is prepared without any reference to, oruse of, the current period’s budget and the likely operating resul ts for the current period. In zero basedbudgeting the manager must start from scratch and justify all incomes and expenses proposed.

C. Performance budgets use statements of missions, goals and objectives to explain why the money is being spent.It is a way to allocate resources to achieve specific objectives based on program goals and measured results.Performance budgeting starts with a goal, then determines strategies and activities to accomplish that goal andustify the expenses. However, that justification is limited to the activities that accomplish a particular goal and does

not extend to the entire budget.

D. Program budgeting is used mainly by non-profit organizations and governmental bodies. It involves planning andbudgeting for a specific program which may have not only expenses but income as well. If the program has income,the goal is to have the program income not only cover the program expenses but to exceed them. Programbudgeting is limited to the activities that relate to one specific program and does not extend to the entire budget.

Question 75 - CMA 1293 H1 - Budget Methodologies

The use of standard costs in the budgeting process signifies that an organization has most likely implemented a

 A. Static budget.B. Flexible budget.

Part 1 : 07/28/10 08:28:23

(c) HOCK international, page 45

Page 46: Budget Methodologies

8/20/2019 Budget Methodologies

http://slidepdf.com/reader/full/budget-methodologies 46/61

C. Capital budget.D. Zero-base budget.

 A.

 A fixed budget, or static budget, is a budget that is prepared for only one level of activity within the company. A staticbudget does not require the use of standard costs.

B. A flexible budget is a budget that is prepared for the actual activity level achieved during the period. Thisis done us ing the standard cost per unit. Therefore, if a company is using standard costs it may indicate thatthey are also using a flexible budget.

C.

The capital budget is the budget in which all capital (property, plant and equipment) expenditures are planned. Thisbudget is often prepared years in advance so that the company is able to obtain the necessary financing oraccumulate the necessary cash to carry out its capital expansion plans. Although capital expansion plans that affectthe budget being developed must be incorporated into that budget, there is no reason to say that use of standardcosts in the budgeting process signifies that an organization has implemented a capital budget. To the extent that thecapital budget is incorporated into the budget being developed, a capital budget is a part of any budget, whether it is

a static budget, a flexible budget, or a zero-base budget.

D. Zero-based budgeting is the budgeting method in which the current year budget is prepared without any referenceto, or use of, the prior period's budget. The use of this form of budgeting does not require the use of standard costs.

Question 76 - CMA 692 H4 - Budget Methodologies

The budget that describes the long-term position, goals, and objectives of an entity within its environment is the

 A. Operating budget.

B. Capital budget.C. Cash management budget.D. Strategic budget.

 A. The operating budget is a short-term budget that is related to the operations of the company. It does not describethe long-term position, goals and objectives of the company within its environment.

B. The capital budget is used to plan all capital expenditures (property, plant and equipment). This budget is oftenprepared years in advance, so that the company can plan to obtain the necessary financing or accumulate theneeded cash to carry out its capital expansion plans. However, it does not describe the long-term position, goals andobjectives of an entity within its environment.

C. The cash management budget relates to the cash inflows, outflows and cash balances of the company. It is a

short-term budget that is concerned with l iquidity. It does not describe the long-term position, goals and objectives ofthe company within its environment.

D. Long-term, or strategic, planning for periods o f five years and more which is based on the objectives ofthe organization is called the strategic plan.

Question 77 - CMA 1292 H5 - Budget Methodologies

Barnes Corporation expected to sell 150,000 board games during the month of November, and the company'smaster budget contained the following data related to the sale and production of these games:

Revenue $2,400,000

Direct materials 675,000

Part 1 : 07/28/10 08:28:23

(c) HOCK international, page 46

Page 47: Budget Methodologies

8/20/2019 Budget Methodologies

http://slidepdf.com/reader/full/budget-methodologies 47/61

Direct labor 300,000

Variable overhead 450,000

Contribution $ 975,000

Fixed overhead 250,000

Fixed selling/administration 500,000

Operating income $ 225,000

 Actual sales during November were 180,000 games. Using a flexible budget, the company expects the operatingincome for the month of November to be

 A. $270,000.B. $510,000.C. $225,000.D. $420,000.

 A. This is calculated using the static budget operating income per unit of $1.50 ($225,000 ÷ 150,000) and multipliedby the actual level of sales of 180,000. This gives the operating income of $270,000 ($1.5 × 180,000), but it ignoresthe fact that fixed costs do not vary with the level of activity.

B. This answer results from incorrectly treating variable overhead costs as fixed costs. See the correct answer for acomplete explanation.

C. This is the static budget operating income not adjusted for the actual level of output.

D.

The flexible budget is the budget developed for the actual achieved level of activity rather than the masterbudget level. To compute the flexible budget we must use the standard cos ting system, i.e. we need todetermine the budgeted selling price, budgeted variable cost per unit, and budgeted total amount of fixedcosts.

The budgeted selling price minus the budgeted cost per unit equal the contribution per unit, so we can use

the budgeted contribution per unit and need not calculate the first tw o items. The budgeted contribution perunit is $6.50 ($975,000 ÷ 150,000). Fixed costs for the flexible budget are the same as for the static budget,since fixed costs do not fluctuate with the level of output.

The flexible budget contribu tion margin for sales of 180,000 is $1,170,000 ($6.50 × 180,000). Subtracting thefixed costs from the expected contribution margin fo r the actual level of sales, we arrive at a flexible budgetoperating income of $420,000 ($1,170,000  $250,000  $500,000 = $420,000).

Question 78 - IMA 08-P2-16 - Budget Methodologies

 All of the following are disadvantages of top-down budgeting as opposed to participatory budgeting, except  that it:

 A. Reduces the time required for budgeting.B. May result in a budget that is not possible to achieve.C. Reduces the communication between employees and management.D. May limit the acceptance of proposed goals and objectives.

 A. This is not a disadvantage o f top-down budget ing. A top dow n budget does not requi re as muchdevelopment time, as fewer voices must be heard.

B. This is a disadvantage of top-down budgeting. Senior management may decree a desired outcome that isunrealistic under current working conditions. Senior management is not generally as familiar with day to day tasksand constraints as lower level managers and employees would be.

C. This is a disadvantage of top-down budgeting. A participatory budget process provides management at all levelsan opportunity to solicit ideas from employees. Without this approach, communication between employees and

Part 1 : 07/28/10 08:28:23

(c) HOCK international, page 47

Page 48: Budget Methodologies

8/20/2019 Budget Methodologies

http://slidepdf.com/reader/full/budget-methodologies 48/61

management is reduced, as employees may not feel as open to voicing opinions.

D. This is a disadvantage of top-down budgeting. When senior management dictates the budget, lower levelmanagers and their staff may not only not accept the budget, but they may also not accept senior management’sgoals and objectives on which the budget is based. Senior management may very well be unaware of opportunitiesand strengths within their divisions. Often these opportunities are introduced by divisional management personnelwho can see from the ground level how an opportunity can be capitalized upon. These leads are often unknown tosenior management.

Question 79 - CMA 1293 3-12 - Budget Methodologies

Superflite expects April sales of its deluxe model airplane, the C-14, to be 402,000 units at $11 each. Each C-14requires three purchased components shown below.

 Purchase

CostNumber Needed

for each C-14 Unit

 A-9 $0.50 1B-6 0.25 2

D-28 1.00 3

Factory direct labor and variable overhead per unit of C-14 totals $3.00. Fixed factory overhead is $1.00 per unit at aproduction level of 500,000 units. Superflite plans the following beginning and ending inventories for the month of

 April and uses standard absorption costing for valuing inventory.

Part No.Units at April 1

Units at April 30

C-14 12,000 10,000

 A-9 21,000 9,000

B-6 32,000 10,000D-28 14,000 6,000

 Assume Superflite plans to manufacture 400,000 units in April. The total April budget for all purchased componentsshould be

 A. $1,596,500.B. $1,608,500.C. $1,600,000.D. $1,580,500.

 A. This answer is incorrect. See the correct answer for a complete explanation.

B. This answer is incorrect. See the correct answer for a complete explanation.

C. This answer is incorrect. See the correct answer for a complete explanation.

D.

To solve these question we should use the formula for the phys ical flow of inventory to calculate theamount to be purchased for each component (A-9, B-6 and D-28), then use the price of each component tocalculate the total amount to be budgeted for April's produc tion.

Beginning Inventory + Units Purchased  Units Used in Production = Ending Inventory

For A-9: one unit of A-9 is necessary for each unit o f finished produc t. Plugging numbers fo r A-9 into the

formula we will get the following: 21,000 + Units Purchased  (400,000 × 1) = 9,000. Solving for UnitsPurchased, we get Unit s Purchased = 388,000. The cost to purchase 388,000 units o f A-9 $194,000 ($0.50 ×388,000).

Part 1 : 07/28/10 08:28:23

(c) HOCK international, page 48

Page 49: Budget Methodologies

8/20/2019 Budget Methodologies

http://slidepdf.com/reader/full/budget-methodologies 49/61

For B-6: two un its of B-6 are necessary for each unit of finished product. Plugging numbers for B-6 into theformula we will get the following: 32,000 + Units Purchased  (400,000 × 2) = 10,000. Solving for UnitsPurchased, we get Units Purchased = 778,000. The cost to purchase 778,000 units of B-6 is $194,500 ($0.25× 778,000).

For D-28: three units o f D-28 are necessary for each un it of fin ished product. Plugging numbers for D-28 intothe formula we get the follow ing: 14,000 + Units Purchased  (400,000 × 3) = 6,000. Solv ing for UnitsPurchased, we get Unit s Purchased = 1,192,000. The cost to purchase 1,192,000 units o f D-28 is $1,192,000($1.00 × 1,192,000).

 Add ing the cos ts to purchase all o f the components together , we get $1,580,500 ($194,000 + $194,500 +$1,192,000).

Question 80 - CIA 593 IV-12 - Budget Methodologies

 A company has the following budget data:

Beginning finished goods inventory 40,000 units

Sales 70,000 units

Ending finished goods inventory 30,000 units

Direct materials $10 per unit

Direct labor $20 per unit

Variable factory overhead $5 per unit

Selling costs $2 per unit

Fixed factory overhead $80,000

What will be the total budgeted production costs?

 A. $2,100,000B. $2,300,000C. $2,180,000D. $2,220,000

 A. This is the total variable costs of production.

B. In this answer it incorrectly assumes that sell ing costs are a variable production cost. However, selling costs are aperiod cost.

C.

First, we need to determine the number of units produced in during the period using the formula for thephysical flow of goods : Beginning Inventory + Units Produced  Units Sold = Ending Inventory.

40,000 + Units Produced  70,000 = 30,000.Units Produced = 60,000

We then multiply the unit variable production cos t by the number of units produced and add the product tothe fixed overhead cost, since fixed overhead is a production cost also. Period costs such as selling costsare not production costs and we do not take them into consideration when calculating the budgetedproduction costs.

Unit variable production costs are: $10 DM + $20 DL + $5 VOH = $35. Total variable production costs are:$35 × 60,000 = $2,100,000. Total p roduction cos ts are: $2,100,000 + $80,000 = $2,180,000

D. In this answer it incorrectly assumes that selling costs are variable production costs, and fixed overhead are notincluded in calculation. However, selling costs are a period cost, not a production cost, and fixed overheads are a

Part 1 : 07/28/10 08:28:23

(c) HOCK international, page 49

Page 50: Budget Methodologies

8/20/2019 Budget Methodologies

http://slidepdf.com/reader/full/budget-methodologies 50/61

production cost.

Question 81 - CMA 1296 H12 - Budget Methodologies

Daffy Tunes manufactures a toy rabbit with moving parts and a built-in voice box. Projected sales in units for the next5 months are as follows:

Month Sales in Units

January 30,000

February 36,000

March 33,000

 April 40,000

May 29,000

Each rabbit requires basic materials that Daffy purchases from a single supplier at $3.50 per rabbit. Voice boxes arepurchased from another supplier at $1.00 each. Assembly labor cost is $2.00 per rabbit, and variable overhead costis $.50 per rabbit. Fixed manufacturing overhead applicable to rabbit production is $12,000 per month. Daffy's policyis to manufacture 1.5 times the coming month's projected sales every other month, starting with January (i.e.,odd-numbered months) for February sales, and to manufacture 0.5 times the coming month's projected sales inalternate months (i.e., even-numbered months). This allows Daffy to allocate limited manufacturing resources toother products as needed during the even-numbered months.

The unit production budget for toy rabbits for January is

 A. 45,000 units.B. 14,500 units.C. 54,000 units.D. 16,500 units.

 A. The level of production needs to be based on the following month projected sales, not on the current month'ssales.

B. This is the budgeted production for April.

C. This is a very long quest ion, but with only a few import ant pieces of information. In January, theproduction wi ll be equal to 1.5 times the expected sales in February. Expected February sales are 36,000, soin January the company wi ll produce 54,000 units.

D. This is the budgeted production for February.

Question 82 - CIA 1193 IV-13 - Budget Methodologies

 A company has budgeted sales for the upcoming quarter as follows:

  January February March

Units 15,000 18,000 16,500

The ending finished goods inventory for each month equals 50% of the next month's budgeted sales. Additionally, 3pounds of raw materials are required for each finished unit produced. The ending raw materials inventory for eachmonth equals 200% of the next month's production requirements. If the raw materials cost $4.00 per pound andmust be paid for in the month purchased, the budgeted raw materials purchases (in dollars) for January are

 A. $180,000B. $198,000

Part 1 : 07/28/10 08:28:23

(c) HOCK international, page 50

Page 51: Budget Methodologies

8/20/2019 Budget Methodologies

http://slidepdf.com/reader/full/budget-methodologies 51/61

C. $216,000D. $207,000

 A. This is a dollar cost of materials required for January sales ($4.00 * 3 * 15,000).

B. This is a dollar cost of materials required for January production ($4.00 * 3 * 16,500), not for purchases ofmaterials.

C.

First, we need to determine the production requirements for January and February by using the formula forthe physical flow of finished goods : Beginning Inventory + Units Produced  Units Sold = Ending Inventory.

Beginning Inventory for January wi ll be 50% of January's sales, or .50 × 15,000. Ending Inventory fo rJanuary w ill be 50% of February's sales, or .50 × 18,000. Therefore, the formula for January production is:

7,500 + Units Produced  15,000 = 9,000Units Produced = 16,500

Beginning Inventory for February will be 50% of February's sales, already calculated as 9,000. Ending

Inventory fo r February wi ll be 50% of March's sales, or .50 × 16,500. The formula for February production is:

9,000 + Units Produced  18,000 = 8,250Units Produced = 17,250

We know that th ree pounds of raw materials are needed for each unit, and there were no work-in-processbalances in either month. Now, we can determine the beginning and ending inventor y level of raw materialsfor January and the usage for January.

Beginning inventory for January is 99,000 lb. (16,500 * 3 * 200%). Ending inventory for January is 103,500 lb.(17,250 * 3 * 200%). The amount of r aw materials used dur ing January is the January production of 16,500 * 3pounds per un it, or 49,500 pounds.

Plugging these numbers into the formula of t he physical flow of goods w e will get the following:

99,000 + Purchases  49,500 = 103,500Purchases = 54,000 pounds of raw material to be purchased in January

Multiplying this quantity by the price per pound of raw material we will get the dollar amount of purc hases inJanuary: $216,000 ($4.00 * 54,000).

D. This is a dollar cost of materials required for February production ($4.00 * 3 * 17,250).

Question 83 - CMA 1292 H4 - Budget Methodologies

 A budget manual, which enhances the operation of a budget system, is most likely to include

 A. Employee hiring policies.B. Distribution instructions for budget schedules.C. A chart of accounts.D. Documentation of the accounting system software.

 A. Employee hiring policies should be included in the personnel manual, and they are not necessary for budgetpreparation.

B. A budget manual details the budget process. One of the areas that must be included in the budget manual

is the communication and distribution p rocess. As one budget is completed it must be sent to all thedepartments whose budgets are based on that already completed budget.

Part 1 : 07/28/10 08:28:23

(c) HOCK international, page 51

Page 52: Budget Methodologies

8/20/2019 Budget Methodologies

http://slidepdf.com/reader/full/budget-methodologies 52/61

C. A chart of accounts is included in the accounting manual, not the budgeting manual.

D. Documentation of the accounting system software is not needed for budget preparation and is not included in thebudget manual.

Question 84 - CIA 598 3-21 - Budget Methodologies

 A flexible budget is a quantitative expression of a plan that:

 A. Projects costs on the basis of future improvements in existing practices and procedures during a budget period.B. Is developed for the actual level of output achieved for the budget period.C. Focuses on the costs of activities necessary to produce and sell products and services for a budget period.D. Is comprised of the budgeted income statement and its supporting schedules for a budget period.

 A. This is the definition of Kaizen budgeting.

B. This is the definition of a flexible budget.

C. This is the definition of activity-based budgeting.

D. This is the definition of an operating budget.

Question 85 - CMA 1294 3-8 - Budget Methodologies

Super Drive, a computer disk storage and back-up company, uses accrual accounting. The company's Statement ofFinancial Position for the year ended November 30, is as fol lows:

Super DriveStatement of Financial Position

November 30

 Assets

Cash $52,000

 Accounts receivable, net. 150,000

Inventory 315,000

Property, plant and equipment 1,000,000

Total assets $1,517,000

Liabilities and Equity 

 Accounts payable $175,000

Common stock 900,000Retained earnings 442,000

Total liabilities and shareholders equity $1,517,000

 Additional information regarding Super Drive's operations include the following:

Sales are budgeted at $520,000 for December and $500,000 for January of the next year.

Collections are expected to be 60% in the month of sale and 40% in the month following the sale.

80% of the disk drive components are purchased in the month prior to the month of sale, and 20% arepurchased in the month of sale. Purchased components are 40% of the cost of goods sold.

Payment for the components is made in the month following the purchase.

Cost of goods sold is 80% of sales.

The projected balance in accounts payable on December 31 is

Part 1 : 07/28/10 08:28:23

(c) HOCK international, page 52

Page 53: Budget Methodologies

8/20/2019 Budget Methodologies

http://slidepdf.com/reader/full/budget-methodologies 53/61

 A. $166,400B. $326,400C. $416,000D. $161,280

 A. This is the cost of components in the cost of goods sold in December ($416,000 * 40% = $166,400). See thecorrect answer for a complete explanation.

B. This answer is incorrect. See the correct answer for a complete explanation.

C. This is a cost of goods sold in December. See the correct answer for a complete explanation.

D.

Purchases in December will affect the end of December balance of account s payable, as all payments aremade in the month fol lowing purchase. Purchases made in November will no t have an effect on December

 A/P, as they w il l have been fu lly paid during December. Thus, we need t o determine the amount ofpurchases in December.

Purchased components are 40% of the cos t of goods sold and cost of goods sold is 80% of sales.

Purchases in December are 80% of components fo r January sales, and 20% for December sales. The cost ofgoods sold in January w ill be $400,000 ($500,000 × 80%). The cost o f components in cos t of goods sold inJanuary is $160,000 ($400,000 × 40%). Purchases of components to satisfy January sales made in Decemberare $128,000 ($160,000 × 80%). The cost of goods sold in December is $416,000 ($520,000 × 80%). The costof components in cost of goods sold in December is $166,400 ($416,000 × 40%). Purchases of componentsto satisfy December sales made in December are $33,280 ($166,400 × 20%). Adding these two numberstogether we get the amount of Accounts Payable for December of $161,280 ($128,000 + $33,280).

Question 86 - CMA 692 3-25 - Budget Methodologies

Berol Company plans to sell 200,000 units of finished product in July and anticipates a growth rate in sales of 5% permonth. The desired monthly ending inventory in units of finished product is 80% of the next month's estimated sales.There are 150,000 finished units in inventory on June 30. Each unit of finished product requires 4 pounds of directmaterials at a cost of $1.20 per pound. There are 800,000 pounds of direct materials in inventory on June 30.

Berol Company's production requirement in units of finished product for the 3-month period ending September 30 is

 A. 712,025 units.B. 665,720 units.C. 638,000 units.D. 630,500 units.

 A. This answer is incorrect. See the correct answer for a complete explanation.

B.

This question covers a three month period beginning July 1 and ending September 30. Beginning inventoryin July is 150,000 finished units. The desired monthly ending inventor y in units o f finished product is 80% ofthe next month 's estimated sales. According to the expected sales growth rate, the company plans to sell210,000 units (200,000 × 105%) in August, 220,500 unit s (210,000 × 105%) in September and 231,525 units(220,500 × 105%) in October. Thus, ending inventory in September wil l need to be 185,220 units (231,525 ×80%). During July, August and September sales are forecasted to be 630,500 units (200,000 + 210,000 +220,500). Now we can calcu late the number of units that need to be produced:

Beginning Inventory + Units Produced  Units Sold = Ending Inventory

150,000 + Units Produced

 630,500 = 185,220.

Solving for Units Produced,

Part 1 : 07/28/10 08:28:23

(c) HOCK international, page 53

Page 54: Budget Methodologies

8/20/2019 Budget Methodologies

http://slidepdf.com/reader/full/budget-methodologies 54/61

Units Produced = 665,720.

C. This answer is incorrect. See the correct answer for a complete explanation.

D. This is the expected sales for the next three months. See the correct answer for a complete explanation.

Question 87 - CMA 1283 4-25 - Budget Methodologies

Kelly Company is a retail sporting goods store that uses accrual accounting for its records. Facts regarding Kelly'soperations are as follows:

Sales are budgeted at $220,000 for December year 1 and $200,000 for January year 2.

Collections are expected to be 60% in the month of sale and 38% in the month following the sale.

Gross margin is 25% of sales.

 A total of 80% of the merchandise held for resale is purchased in the month prior to the month of sale and20% is purchased in the month of sale. Payment for merchandise is made in the month following the

purchase.Other expected monthly expenses to be paid in cash are $22,600.

 Annual depreciation is $216,000.

Below is Kelly Company's statement of financial position at November 30, year 1.

 Assets  

Cash $22,000

 Accounts receivable(net of $4,000 allowance for uncollectible accounts)

76,000

Inventory 132,000

Property, plant, and equipment (net of $680,000 accumulated deprecation) 870,000

Total assets $1,100,000

Liabilities and Stockholders' Equity  

 Accounts payable $162,000

Common stock 800,000

Retained earnings 138,000

Total liabilities and stockholders' equity $1,100,000

The projected balance in inventory on December 31, year 1 is

 A. $153,000.B. $120,000.C. $160,000.D. $150,000.

 A. This is the total purchases to be made during the month of December, but it is not the ending inventory balance.Ending inventory is equal to beginning inventory + purchases made during the period  cost of inventory sold duringthe period. See the correct answer for a complete explanation.

B.

Ending inventory is equal to beginning inventory + purchases made during the period  cost of inventorysold during the period.

Beginning inventory is given as $132,000.

Purchases during December are 80% of the cost of inventory projected to be sold during January and 20%of the cost of i nventory projected to be sold dur ing December. Since the gross margin is 25%, the projectedcost o f sales will be 75% of projected sales. Sales are pro jected at $220,000 for December and $200,000 for

Part 1 : 07/28/10 08:28:23

(c) HOCK international, page 54

Page 55: Budget Methodologies

8/20/2019 Budget Methodologies

http://slidepdf.com/reader/full/budget-methodologies 55/61

January. Therefore, total purchases du ring December will be (January sales of $200,000 × .75 × .80) +(December sales of $220,000 × .75 × .20), or $120,000 + $33,000, which equals $153,000.

The cost of inventory sold during the period is December sales of $220,000 × .75, which is $165,000.

Therefore, the ending inventory as of December 31 is:

$132,000 + $153,000  $165,000 = $120,000

C. This is the 80% of projected January sales. Ending inventory is equal to beginning inventory + purchases madeduring the period  cost of inventory sold during the period. See the correct answer for a complete explanation.

D. This is 75% of projected January sales. Ending inventory is equal to beginning inventory + purchases made duringthe period  cost of inventory sold during the period. See the correct answer for a complete explanation.

Question 88 - CMA 1295 H6 - Budget Methodologies

Individual budget schedules are prepared to develop an annual comprehensive or master budget. The budgetschedule that would provide the necessary input data for the direct labor budget would be the

 A. Raw materials purchases budget.B. Sales forecast.C. Schedule of cash receipts and disbursements.D. Production budget.

 A. The raw materials purchases budget does not provide information for the direct labor budget preparation.

B. Sales forecast will give information to determine the level of production. However, it does not provide directinformation to prepare the direct labor budget.

C. The cash budget is the last budget in the master budget, and the direct labor budget must be prepared before thecash budget can be prepared.

D. Once the sales budget has been developed, it provides information w ith which to prepare the product ionbudget. The production budget determines how muchof t he product will be produced. Thus, it providesinformation to prepare the direct labor, direct materials and manufacturing overhead budgets.

Question 89 - CMA 1283 4-22 - Budget Methodologies

Kelly Company is a retail sporting goods store that uses accrual accounting for its records. Facts regarding Kelly's

operations are as follows:Sales are budgeted at $220,000 for December year 1 and $200,000 for January year 2.

Collections are expected to be 60% in the month of sale and 38% in the month following the sale.

Gross margin is 25% of sales.

 A total of 80% of the merchandise held for resale is purchased in the month prior to the month of sale and20% is purchased in the month of sale. Payment for merchandise is made in the month following thepurchase.

Other expected monthly expenses to be paid in cash are $22,600.

 Annual depreciation is $216,000.

Below is Kelly Company's statement of financial position at November 30, year 1.

 Assets  

Cash $22,000

Part 1 : 07/28/10 08:28:23

(c) HOCK international, page 55

Page 56: Budget Methodologies

8/20/2019 Budget Methodologies

http://slidepdf.com/reader/full/budget-methodologies 56/61

 Accounts receivable(net of $4,000 allowance for uncollectible accounts)

76,000

Inventory 132,000

Property, plant, and equipment (net of $680,000 accumulated deprecation) 870,000

Total assets $1,100,000

Liabilities and Stockholders' Equity  

 Accounts payable $162,000

Common stock 800,000

Retained earnings 138,000

Total liabilities and stockholders' equity $1,100,000

The budgeted cash collections for December year 1 are

 A. $212,000.B. $132,000.C. $208,000.D. $203,600.

 A. This amount include the allowance for bad debt, which should not be considered. See the correct answer for acomplete explanation.

B. This is the amount of cash collected from December sales in December. See the correct answer for a completeexplanation.

C. Cash col lection o f December sales in December is 60% of the sales level, or $132,000 ($220,000 × 60%).December cash col lections also include some of the proceeds from November sales. Accounts receivablenet of bad debt allowance are $76,000 as of November 30, which are going to be collected in their entirety inDecember. Thus, the total budgeted cash collections in December is $208,000 ($132,000 + $76,000).

D. This is the budgeted cash collections for January Year 2. See the correct answer for a complete explanation.

Question 90 - CMA 1295 H4 - Budget Methodologies

When preparing the series of annual operating budgets, management usually starts the process with the

 A. Sales budget.B. Cash budget.C. Balance sheet.D. Capital budget.

 A. The sales budget is usually the f irst budget to be prepared .

B. The cash budget is usually the last budget prepared. See the correct answer for a complete explanation.

C. The balance sheet may be prepared only after all of the individual budgets have been prepared. See the correctanswer for a complete explanation.

D. The capital budget is a long-term budget and is prepared outside of the annual budgeting process. See thecorrect answer for a complete explanation.

Question 91 - IMA 08-P2-47 - Budget Methodologies

Tidwell Corporation sells a single product for $20 per unit. All sales are on account, with 60% collected in the monthof sale and 40% collected in the following month. A partial schedule of cash collections for January through March

Part 1 : 07/28/10 08:28:23

(c) HOCK international, page 56

Page 57: Budget Methodologies

8/20/2019 Budget Methodologies

http://slidepdf.com/reader/full/budget-methodologies 57/61

of the coming year reveals the following receipts for the period.

  Cash Receipts

January February March

December receivables $32,000

From January sales 54,000 $36,000

From February sales 66,000 $44,000

Other information includes the following:

• Inventories are maintained at 30% of the following month's sales.

• Assume that March sales total $150,000.

The number of units to be purchased in February is

 A. 7,750 units.B. 4,900 units.

C. 6,100 units.D. 3,850 units.

 A. This answer assumes that beginning inventory was zero. Beginning inventory should be 30% of February's sales,or 1,650 units.

B. This calculation has all of the numbers correct but it reverses the beginning inventory and ending inventorybalances in the inventory equation. The inventory equation is Beginning Inventory + Purchases  Sales = EndingInventory.

C.

Needed information to be calculated is beginning inventory for February, sales for February and ending

inventor y for February. These are needed in units in o rder to calculate purchases for February. Theinventory equation is Beginning Inventory + Purchases  Sales = Ending Inventory . When any three of theseamounts are available or can be calculated, the fourth amount can always be calculated. This equation canbe used with either monetary amounts or number of units. Here we have monetary amounts and will convertit to number of units us ing the product price of $20 per unit.

The beginning inventory for February w ill depend upon February sales. February sales can be calculatedfrom the cash receipts given as $66,000 + $44,000, or $110,000. $110,000 in sales div ided by the $20 priceper unit = 5,500 units sold during February. Beginning inventory for February will be 30% of 5,500, or 1,650units.

Ending inventory for February will be based on t he number of sales in March, which is $150,000 divided by$20, or 7,500 units. Ending inventory for February wil l be 30% of 7,500, or 2,250 uni ts.

The number of units to be purchased in February, using the inventory equation, will be:

1,650 + X  5,500 = 2,250

X = 6,100 units

D. This answer assumes that ending inventory is zero. Ending inventory should be 30% of March sales, or 2,250.

Question 92 - CMA 691 3-1 - Budget Methodologies

Wilson Company uses a comprehensive planning and budgeting system. The proper order for Wilson to preparecertain budget schedules would be

Part 1 : 07/28/10 08:28:23

(c) HOCK international, page 57

Page 58: Budget Methodologies

8/20/2019 Budget Methodologies

http://slidepdf.com/reader/full/budget-methodologies 58/61

 A. Cost of goods sold, balance sheet, income statement, and statement of cash flows.B. Statement of cash flows, cost of goods sold, income statement, and balance sheet.C. Income statement, balance sheet, statement of cash flows, and cost of goods sold.D. Cost of goods sold, income statement, balance sheet, and statement of cash flows.

 A. The income statement is an input to the balance sheet, so the income statement is prepared before the balance

sheet.

B. The statement of cash flows is the last financial statement to be prepared in the budgeting process.

C. The budget for cost of goods sold is prepared before the budgeted income statement and before any of f inancialbudgets (capital budget, cash budget, budgeted balance sheet, budgeted statement of cash flows).

D. Cost of goods sold is a part of operating budget which cu lminates in the preparation of the budgetedincome statement. The budgeted income statement is an input to the budgeted balance sheet, and thebudgeted statement of cash flows is the last to be prepared in the budgeting process.

Question 93 - IMA 08-P2-12 - Budget Methodologies

Helen Thomas, Amador Corporation's vice president of planning, has seen and heard it all. She has told thecorporate controller that she is "....very upset with the degree of slack that veteran managers use when preparingtheir budgets."

Thomas has considered implementing some of the following activities during the budgeting process.

1. Develop the budgets by top management and issue them to lower-level operating units.

2. Study the actual revenues and expenses of previous periods in detail.

3. Have the budgets developed by operating units and accept them as submitted by a company-wide budgetcommittee.

4. Share the budgets with all employees as a means to reach company goals and objectives.

5. Use an iterative budgeting process that has several "rounds" of changes initiated by operating units and/orsenior managers.

Which one of these activities should Amador implement in order to best remedy Thomas's concerns, help eliminatethe problems experienced by Amador, and motivate personnel?

 A. 2 and 3.B. 1 only.C. 2 and 4.D. 2, 4 and 5.

 A. Option 2 is correct, but option 3 won't fix the problem of "slack" that Thomas is concerned about.

B. A budget developed by top management and issued to lower level units is unlikely to motivate employees.

C. Option 2 and option 4 will generate a budget and that has employee buy in, but it may not eliminate slack.

D. Options 2, 4 and 5 are the best way to eliminate slack th rough the rounds of changes and the review ofhistorical information. Option 4 will also help generate employee buy in and mot ivate them to help achievethe overall corporate targets.

Question 94 - CMA 686 4-23 - Budget Methodologies

Simson Company's master budget shows straight-line depreciation on factory equipment of $258,000. The masterbudget was prepared at an annual production volume of 103,200 units of product. This production volume isexpected to occur uniformly throughout the year. During September, Simson produced 8,170 units of product, and

Part 1 : 07/28/10 08:28:23

(c) HOCK international, page 58

Page 59: Budget Methodologies

8/20/2019 Budget Methodologies

http://slidepdf.com/reader/full/budget-methodologies 59/61

the accounts reflected actual depreciation on factory machinery of $20,500. Simson controls manufacturing costswith a flexible budget. The flexible budget amount for depreciation on factory machinery for September would be

 A. $19,475.B. $20,500.C. $21,500.D. $20,425.

 A. Depreciation is a fixed cost and does not change with the level of production.

B. This is the actual amount of depreciation that was recorded during the month. The question asks for the flexiblebudget amount for depreciation, not the actual amount.

C. The flexible budget is a budget that is prepared for the actual level of activity ach ieved during the period.Only budgeted variable cos ts are adjusted in a flexible budget. Fixed cos ts are the same in the flexiblebudget as they are in the static budget, because fixed costs remain the same regardless the level of activity.Since depreciation is a fixed cost, the amount of depreciation expense for the flexible budget is equal to thedepreciation expense for the static budget, or $21,500 (the to tal budgeted for the year of $258,000 ÷ 12).

D. Depreciation is a fixed cost and does not change with the level of production.

Question 95 - IMA 08-P2-02 - Budget Methodologies

When compared to static budgets, flexible budgets:

 A. Offer managers a more realistic comparison of budget and actual fixed cost items under their control.B. Encourage managers to use less fixed costs items and more variable cost items that are under their control.C. Offer managers a more realistic comparison of budget and actual revenue and cost items under their control.D. Provide a better understanding of the capacity variances during the period being evaluated.

 A. A flexible budget takes the variable revenues and costs as they are planned in the master budget and adjusts themaster budget amounts to what they would have been if the actual volume achieved had been used in preparing thebudget. Flexible budgets show how variable costs change at different production levels. Total budgeted fixed costsare the same in both the static budget and the flexible budget, since fixed costs are not dependent in total uponcapacity usage. Since fixed costs are often determined before the budget is developed, budgeted fixed costs aregenerally fairly realistic.

B. Fixed and variable costs are not interchangeable within the organization. Simply using more raw materials will noteliminate the need to pay rent, for example.

C. A flexible budget takes the variable revenues and costs as they are planned in the master budget andadjusts the master budget amounts to what they would have been if the actual volume achieved had beenused in preparing the budget. A flexible budget prepared in addition to the master budget that is used

exclusively for reporting on variances other than those due to volume differences allows management tofocus on the variances that may be caused by p roduction or administrative problems that need attention.

D. Variance analysis, and specifically the fixed overhead production-volume variance, measures the variance due tothe actual production level being different from the production capacity level used to calculate the budgeted fixedoverhead cost to be applied to each unit produced. The production-volume variance, not the flexible budget,provides a better understanding of the result of this difference. Total budgeted fixed costs are the same in both thestatic budget and the flexible budget, since fixed costs are not dependent in total upon capacity usage.

Question 96 - CIA 594 III-68 - Budget Methodologies

 A company produces a product that requires 2 pounds of a raw material. The company forecasts that there will be6,000 pounds of raw material on hand at the end of June. At the end of any given month the company wishes to

Part 1 : 07/28/10 08:28:23

(c) HOCK international, page 59

Page 60: Budget Methodologies

8/20/2019 Budget Methodologies

http://slidepdf.com/reader/full/budget-methodologies 60/61

have 30% of next month's raw material requirements on hand. The company has budgeted production of theproduct for July, August, September, and October to be 10,000, 12,000, 13,000, and 11,000 units, respectively. Asof June 1, the raw material sells for $1.00 per pound.

The cost of inventory is determined using the last-in-first-out (LIFO) method. If the price of raw material increases10% as of June 30, what will be the effect of this increase on the cost of purchases from July to September?

 A. $3,230 increase.B. $60 increase.C. $600 increase.D. $7,060 increase.

 A. This answer is incorrect. See the correct answer for a complete explanation.

B. This answer is incorrect. See the correct answer for a complete explanation.

C. This answer is incorrect. See the correct answer for a complete explanation.

D.

We need to determine the beginning raw materials inventory, the ending raw materials inventory and theamount of raw materials needed for production, so we can determine how muc h raw material will need to bepurchased for the given production period of July through September.

Beginning inventory of raw materials as of Ju ly 1 needs to be 30% of Ju ly's production requirements: 10,000× 2 pounds per unit × 30% = 6,000 lb. The question t ells us that the company forecasts t hat there will be6,000 pounds of raw material on hand at the end of June, so the beginn ing invento ry for July w ill besuff icient (i.e., no additional raw materials will need to be purchased to adjust t he beginning inventory).Ending inventory of raw materials for September needs to be 30% of October's production requ irements:11,000 × 2 pounds per unit × 30% = 6,600 lb.

The number of units to be produced from Ju ly th rough September is 35,000 units (10,000 + 12,000 + 13,000).The amount of raw materials required for production in these three months is 70,000 lb. (35,000 × 2 pounds

per unit). Now we can calculate the amount of materials that the company w ill need to purchase duringthese three months. The standard calculation for inventor y is as follows, and if we know three of the fournumbers, we can always find the fourth one:

Beginning Inventory 6,000

Plus: Purchases ?

Minus: Materials Used   70,000

Equals: Ending Inventory 6,600

We can solve this w ith an algebraic equation, or we can simply " back into" the missing purchases number:

6,000 + X  70,000 = 6,600X  64,000 = 6,600

X = 70,600

We know that the dif ference in the price of materials purchased before June 30th and after June 30th is$0.10 ($1.10  $1.00). Thus, the effect of p rice increase for raw materials fo r the given period is $7,060 ($0.10× 70,600).

Question 97 - IMA 08-P2-34 - Budget Methodologies

Which one of the following best describes the order in which budgets should be prepared when developing theannual master operating budget?

 A. Production budget, revenue budget, direct material budget.B. Production budget, direct material budget, revenue budget.

Part 1 : 07/28/10 08:28:23

(c) HOCK international, page 60

Page 61: Budget Methodologies

8/20/2019 Budget Methodologies

http://slidepdf.com/reader/full/budget-methodologies 61/61

C. Revenue budget, production budget, direct material budget.D. Revenue budget, direct material budget, production budget.

 A. Until the revenue budget and an estimate of how much will be sold has been prepared, the production budgetcannot be developed.

B. Until the revenue budget and an estimate of how much will be sold has been prepared, the production budgetcannot be developed.

C. The revenue budget includes projected sales, which determines how much shou ld be produced; and theamount to be produced determines how much direct material will be needed.

D. Until the production budget has been prepared, the direct material budget cannot be prepared.

Question 98 - IMA 08-P2-32 - Budget Methodologies

What would be the correct chronological order of preparation for the following budgets?

I. Cost of goods sold budget.II. Production budget.III. Purchases budget.IV. Administrative budget.

 A. IV, II, III, IB. II, III, I, IVC. III, II, IV, ID. I, II, III, IV

 A. The administrative budget must come last in this series, as some variable administrative expenses such aspurchasing expenses will be based on the previous budgets.

B. The production budget indicates what needs to be produced and when, so this is information required forthe purchases budget. The COGS budget is based on desired beginning inventory, desired endinginventory, and expected purchases, so the purchases budget must be prepared before the COGS budgetcan be prepared. The administrative budget is last as some variable adminis trative expenses such aspurchasing expenses will be based on the previous budgets.

C. The purchases budget cannot be completed before the production budget is complete.

D. The COGS budget cannot be completed before the production budget and purchases budgets are complete.

Part 1 : 07/28/10 08:28:23